You are on page 1of 80

Theme: Spinal disorders

A. Osteomyelitis
B. Potts disease of the spine
C. Scheuermanns disease
D. Transverse myelitis
E. Tabes dorsalis
F. Subacute degeneration of the cord
G. Brown-Sequard syndrome
H. Syringomyelia
I. Epidural haematoma

Which is the most likely diagnosis for the scenario given. Each option may be used
once, more than once or not at all.

1. A 68 year old man presents to the plastics team with severe burns to his hands.
He is not distressed by the burns. He has bilateral charcot joints. On
examination there is loss of pain and temperature sensation of the upper limbs.

You answered Subacute degeneration of the cord

The correct answer is Syringomyelia

This patient has syringomyelia which selectively affects the spinotholamic


tracts.Syringomyelia is a disorder in which a cystic cavity forms within the
spinal cord. The commonest variant is the Arnold- Chiari malformation in
which the cavity connects with a congenital malformation affecting the
cerebellum. Acquired forms of the condition may occur as a result of previous
meningitis, surgery or tumours. Many neurological manifestations have been
reported, although the classical variety spares the dorsal columns and medial
lemniscus and affecting only the spinothalamic tract with loss of pain and
temperature sensation. The bilateral distribution of this patients symptoms
would therefore favor syringomyelia over SCID or Brown Sequard syndrome.
Osteomyelitis would tend to present with back pain and fever in addition to any
neurological signs. Epidural haematoma large enough to produce neurological
impairment will usually have motor symptoms in addition to any selective
sensory loss, and the history is usually shorter.

2. A 24 year old man presents with localised spinal pain over 2 months which is
worsened on movement. He is known to be an IVDU. He has no history
suggestive of tuberculosis. The pain is now excruciating at rest and not
improving with analgesia. He has a temperature of 39 oC.

Osteomyelitis
In an IVDU with back pain and pyrexia have a high suspicion for
osteomylelitis. The most likely organism is staph aureus and the cervical spine
is the most common region affected. TB tends to affect the thoracic spine and in
other causes of osteomyelitis the lumbar spine is affected.

3. A 22 year man is shot in the back, in the lumbar region. He has increased tone
and hyper-reflexia of his right leg. He cannot feel his left leg.

Brown-Sequard syndrome

Theme from January 2012 exam


Brown -Sequard syndrome is caused by hemisection of the spinal cord. It may
result from stab injuries or lateral vertebral fractures. It results in ipsilateral
paralysis (pyramidal tract) , and also loss of proprioception and fine
discrimination (dorsal columns). Pain and temperature sensation are lost on the
contra-lateral side. This is because the fibres of the spinothalamic tract have
decussated below the level of the cord transection.

Spinal disorders

Dorsal column lesion  Loss vibration and proprioception


 Tabes dorsalis, SACD

Spinothalamic tract  Loss of pain, sensation and temperature


lesion
Central cord lesion  Flaccid paralysis of the upper limbs

Osteomyelitis  Normally progressive


 Staph aureus in IVDU, normally cervical region
affected
 Fungal infections in immunocompromised
 Thoracic region affected in TB

Infarction spinal cord  Dorsal column signs (loss of proprioception and fine
discrimination

Cord compression  UMN signs


 Malignancy
 Haematoma
 Fracture

Brown-sequard  Hemisection of the spinal cord


syndrome  Ipsilateral paralysis
 Ipsilateral loss of proprioception and fine
discrimination
 Contralateral loss of pain and temperature
Image sourced from Wikipedia

Image sourced from Wikipedia

Dermatomes

 C2 to C4 The C2 dermatome covers the occiput and the top part of the neck.
C3 covers the lower part of the neck to the clavicle. C4 covers the area just
below the clavicle.
 C5 to T1 Situated in the arms. C5 covers the lateral arm at and above the
elbow. C6 covers the forearm and the radial (thumb) side of the hand. C7 is
the middle finger, C8 is the lateral aspects of the hand, and T1 covers the
medial side of the forearm.
 T2 to T12 The thoracic covers the axillary and chest region. T3 to T12 covers
the chest and back to the hip girdle. The nipples are situated in the middle of
T4. T10 is situated at the umbilicus. T12 ends just above the hip girdle.
 L1 to L5 The cutaneous dermatome representing the hip girdle and groin area
is innervated by L1 spinal cord. L2 and 3 cover the front part of the thighs. L4
and L5 cover medial and lateral aspects of the lower leg.
 S1 to S5 S1 covers the heel and the middle back of the leg. S2 covers the back
of the thighs. S3 cover the medial side of the buttocks and S4-5 covers the
perineal region. S5 is of course the lowest dermatome and represents the skin
immediately at and adjacent to the anus.

Myotomes

Upper limb
Elbow flexors/Biceps C5
Wrist extensors C6
Elbow extensors/Triceps C7
Long finger flexors C8
Small finger abductors T1

Lower limb
Hip flexors (psoas) L1 and L2
Knee extensors (quadriceps) L3
Ankle dorsiflexors (tibialis anterior) L4 and L5
Toe extensors (hallucis longus) L5
Ankle plantar flexors (gastrocnemius) S1

The anal sphincter is innervated by S2,3,4


A 24 year old man is brought to the emergency department have suffered a crush
injury to his forearm. Assessment demonstrates that the arm is tender, red and
swollen. There is clinical evidence of an ulnar fracture and the patient cannot move
their fingers. Which is the most appropriate course of action?

A. Application of an external fixation device

B. Closed reduction

C. Debridement

D. Discharge and review in fracture clinic

E. Fasciotomy
Theme from April 2012
The combination of a crush injury, limb swelling and inability to move digits should
raise suspicion of a compartment syndrome that will require a fasciotomy

Compartment syndrome

 This is a particular complication that may occur following fractures (or


following ischaemia reperfusion injury in vascular patients). It is characterised
by raised pressure within a closed anatomical space.
 The raised pressure within the compartment will eventually compromise tissue
perfusion resulting in necrosis. The two main fractures carrying this
complication include supracondylar fractures and tibial shaft injuries.

Symptoms and signs

 Pain, especially on movement (even passive)


 Parasthesiae
 Pallor may be present
 Arterial pulsation may still be felt as the necrosis occurs as a result of
microvascular compromise
 Paralysis of the muscle group may occur

Diagnosis

 Is made by measurement of intracompartmental pressure measurements.


Pressures in excess of 20mmHg are abnormal and >40mmHg is diagnostic.

Treatment

 This is essentially prompt and extensive fasciotomies


 In the lower limb the deep muscles may be inadequately decompressed by the
inexperienced operator when smaller incisions are performed
 Myoglobinuria may occur following fasciotomy and result in renal failure and
for this reason these patients require aggressive IV fluids
 Where muscle groups are frankly necrotic at fasciotomy they should be
debrided and amputation may have to be considered
 Death of muscle groups may occur within 4-6 hours

Theme: Disorders of the knee

A. Chondromalacia patellae
B. Dislocated patella
C. Undisplaced fracture patella
D. Displaced patella fracture
E. Avulsion fracture of the tibial tubercle
F. Quadriceps tendon rupture
G. Osgood Schlatters disease

Please select the most likely explanation for the scenario given. Each option may be
used once, more than once or not at all.

5. A 19 year old sportswoman presents with knee pain which is worse on walking
down the stairs and when sitting still. On examination there is wasting of the
quadriceps and pseudolocking of the knee.

You answered Dislocated patella

The correct answer is Chondromalacia patellae

A teenage girl with knee pain on walking down the stairs is characteristic for
chondromalacia patellae(anterior knee pain). Most cases are managed with
physiotherapy.

6. A tall 18 year old male athlete is admitted to the emergency room after being hit
in the knee by a hockey stick. On examination his knee is tense and swollen. X-
ray shows no fractures.

You answered Avulsion fracture of the tibial tubercle

The correct answer is Dislocated patella

A patella dislocation is a common cause of haemarthrosis and many will


spontaneously reduce when the leg is straightened. In the chronic setting
physiotherapy is used to strengthen the quadriceps muscles.

7. An athletic 15 year old boy presents with knee pain of 3 weeks duration. It is
worst during activity and settles with rest. On examination there is tenderness
overlying the tibial tuberosity and an associated swelling at this site.

You answered Chondromalacia patellae

The correct answer is Osgood Schlatters disease

Athletic boys and girls may develop this condition in their teenage years. It is
caused by multiple micro fractures at the point of insertion of the tendon into
the tibial tuberosity. Most cases settle with physiotherapy and rest.
Knee injury

Types of injury

Ruptured anterior  Sport injury


cruciate ligament  Mechanism: high twisting force applied to a bent knee
 Typically presents with: loud crack, pain and RAPID
swelling knee (haemoarthrosis)
 Poor healing
 Management: intense physiotherapy or surgery

Ruptured posterior  Mechanism: hyperextension injuries


cruciate ligament  Tibia lies back on the femur
 Paradoxical anterior draw test

Rupture of medial  Mechanism: leg forced into valgus via force outside the
collateral ligament leg
 Knee unstable when put into valgus position

Menisceal tear  Rotational sporting injuries


 Delayed knee swelling
 Joint locking (Patient may develop skills to "unlock"
the knee
 Recurrent episodes of pain and effusions are common,
often following minor trauma

Chondromalacia  Teenage girls, following an injury to knee e.g.


patellae Dislocation patella
 Typical history of pain on going downstairs or at rest
 Tenderness, quadriceps wasting

Dislocation of the  Most commonly occurs as a traumatic primary event,


patella either through direct trauma or through severe
contraction of quadriceps with knee streched in valgus
and external rotation
 Genu valgum, tibial torsion and high riding patella are
risk factors
 Skyline x-ray views of patella are required, although
displaced patella may be clinically obvious
 An osteochondral fracture is present in 5%
 The condition has a 20% recurrence rate

Fractured patella  2 types:

i. Direct blow to patella causing undisplaced fragments


ii. Avulsion fracture
Tibial plateau  Occur in the elderly (or following significant trauma in
fracture young)
 Mechanism: knee forced into valgus or varus, but the
knee fractures before the ligaments rupture
 Varus injury affects medial plateau and if valgus
injury, lateral plateau depressed fracture occurs
 Classified using the Schatzker system (see below)

Schatzker Classification system for tibial plateau fractures


Type Features
1- vertical split of lateral Fracture through dense bone, usually in the young. It may
condyle be virtually undisplaced, or the condylar fragment may be
pushed inferiorly and tilted
2- a vertical split of the The wedge fragement (which may be of variable size), is
lateral condyle combined displaced laterally; the joint is widened. Untreated, a
with an adjacent valgus deformity may develop
loadbearing part of the
condyle
3- depression of the The split does not extend to the edge of the plateau.
articular surface with intact Depressed fragments may be firmly embedded in
condylar rim subchondral bone, the joint is stable
4- fragment of the medial Two injuries are seen in this category; (1) a depressed
tibial condyle fracture of osteoporotic bone in the elderly. (2) a high
energy fracture resulting in a condylar split that runs from
the intercondylar eminence to the medial cortex.
Associated ligamentous injury may be severe
5-fracture of both condyles Both condyles fractured but the column of the metaphysis
remains in continuity with the tibial shaft
6-combined condylar and High energy fracture with marked comminution
subcondylar fractures
A 10 year old boy presents with symptoms of right knee pain. The pain has been
present on most occasions for the past three months and the pain typically lasts for
several hours at a time. On examination he walks with an antalgic gait and has
apparent right leg shortening. What is the most likely diagnosis?

A. Perthes Disease

B. Osteosarcoma of the femur

C. Osteoarthritis of the hip

D. Transient synovitis of the hip

E. Torn medial meniscus

Theme from September 2012 Exam


There are many causes of the irritable hip in the 10-14 year age group. Many of these
may cause both hip pain or knee pain. Transient synovitis of the hip the commonest
disorder but does not typically last for 3 months. An osteosarcoma would not usually
present with apparent limb shortening unless pathological fracture had occurred. A
slipped upper femoral epiphysis can cause a similar presentation although it typically
presents later and with different patient characteristics.

Perthes disease

Perthes disease

 Idiopathic avascular necrosis of the femoral epiphysis of the femoral head


 Impaired blood supply to femoral head, causing bone infarction. New vessels
develop and ossification occurs. The bone either heals or a subchondral
fracture occurs.

Clinical features

 Males 4x's greater than females


 Age between 2-12 years (the younger the age of onset, the better the
prognosis)
 Limp
 Hip pain
 Bilateral in 20%

Diagnosis
Plain x-ray, Technetium bone scan or magnetic resonance imaging if normal x-ray
and symptoms persist.

Catterall staging
Stage Features
Stage 1 Clinical and histological features only
Stage 2 Sclerosis with or without cystic changes and preservation of the articular
surface
Stage 3 Loss of structural integrity of the femoral head
Stage 4 Loss of acetabular integrity

Management

 To keep the femoral head within the acetabulum: cast, braces


 If less than 6 years: observation
 Older: surgical management with moderate results
 Operate on severe deformities

Prognosis
Most cases will resolve with conservative management. Early diagnosis improves
outcomes.
Which of the following types of growth plate fractures may have similar radiological
appearances?

A. Salter Harris types 1 and 5

B. Salter Harris types 4 and 5

C. Salter Harris types 3 and 5

D. Salter Harris types 1 and 2

E. Salter Harris types 1 and 3

Salter Harris injury types 1 and 5 (transverse fracture through growth plate Vs.
Compression fracture) may mimic each other radiologically. Type 5 injuries have the
worst outcomes. Radiological signs of type 5 injuries are subtle and may include
narrowing of the growth plate.

Epiphyseal fractures

Fractures involving the growth plate in children are classified using the Salter - Harris
system.
There are 5 main types.

Salter Harris Classification


Type Description
Type 1 Transverse fracture through the growth plate
Type 2 Fracture through the growth plate to the metaphysis (commonest type)
Type 3 Fracture through the growth plate and the epiphysis with metaphysis spared
Type 4 Fracture involving the growth plate, metaphysis and epiphysis
Type 5 Compression fracture of the growth plate (worst outcome)

Management
Non displaced type 1 injuries can generally be managed conservatively. Unstable or
more extensive injuries will usually require surgical reduction and/ or fixation, as
proper alignment is crucial.
Theme: Pathological fractures

A. Osteosarcoma
B. Osteomalacia
C. Osteoporosis
D. Metastatic carcinoma
E. Osteoblastoma
F. Giant cell tumour
G. Ewing's sarcoma
For each pathological fracture please select the most likely aetiology for the scenario
given. Each option may be used once, more than once or not at all.

2. A 30 year old woman presents with pain and swelling of the left shoulder. There
is a large radiolucent lesion in the head of the humerus extending to the
subchondral plate.

You answered Osteomalacia

The correct answer is Giant cell tumour

Giant cell tumours on x-ray have a 'soap bubble' appearance. They present as
pain or pathological fractures. They commonly metastasize to the lungs.

3. A 72 year old woman has a lumbar vertebral crush fracture. She has
hypocalcaemia and a low urinary calcium.

You answered Osteoporosis

The correct answer is Osteomalacia

Hypocalcemia and low urinary calcium are biochemical features of


osteomalacia. Unfortunately surgeons do need to look at some blood results!

4. A 16 year old boy presents with severe groin pain after kicking a football.
Imaging confirms a pelvic fracture. A previous pelvic x-ray performed 2 weeks
ago shows a lytic lesion with 'onion type' periosteal reaction.

Ewing's sarcoma

A Ewings sarcoma is most common in males between 10-20 years. It can occur
in girls. A lytic lesion with a lamellated or onion type periosteal reaction is a
classical finding on x-rays. Most patients present with metastatic disease with a
5 year prognosis between 5-10%.

Pathological fractures

 A pathological fracture occurs in abnormal bone due to insignificant injury

Causes
Metastatic tumours  Breast
 Lung
 Thyroid
 Renal
 Prostate

Bone disease  Osteogenesis imperfecta


 Osteoporosis
 Metabolic bone disease
 Paget's disease

Local benign conditions  Chronic osteomyelitis


 Solitary bone cyst

Primary malignant tumours  Chondrosarcoma


 Osteosarcoma
 Ewing's tumour

Theme: Shoulder injuries

A. Glenohumeral dislocation
B. Acromioclavicular dislocation
C. Sternoclavicular dislocation
D. Biceps tendon tear
E. Supraspinatus tear
F. Fracture of the surgical neck of the humerus
G. Infra spinatus tear

For each scenario please select the most likely underlying diagnosis. Each option may
be used once, more than once or not at all.

5. A 23 year old rugby player falls directly onto his shoulder. There is pain and
swelling of the shoulder joint. The clavicle is prominent and there appears to be
a step deformity.

Acromioclavicular dislocation

Acromioclavicular joint (ACJ) dislocation normally occurs secondary to direct


injury to the superior aspect of the acromion. Loss of shoulder contour and
prominent classical are key features. NB rotator cuff tears rarely occur in the
second decade.

6. A 22 year old man falls over and presents to casualty. A shoulder x-ray is
performed, the radiologist comments that a Hill-Sachs lesion is present.
You answered Biceps tendon tear

The correct answer is Glenohumeral dislocation

A Hill-Sachs lesion is when the cartilage surface of the humerus is in contact


with the rim of the glenoid. About 50% of anterior glenohumeral dislocations
are associated with this lesion.

7. An 82 year old female presents to A&E after tripping on a step. She complains
of shoulder pain. On examination there is pain to 90o on abduction.

Supraspinatus tear

A supraspinatus tear is the most common of rotator cuff tears. It occurs as a


result of degeneration and is rare in younger adults.

Shoulder disorders

Proximal humerus fractures

 Very common. Usually through the surgical neck. Number of classification


systems though for practical purposes describing the number of fracture
fragments is probably easier. Some key points:
 It is rare to have fractures through the anatomical neck.
 Anatomical neck fractures which are displaced by >1cm carry a risk of
avascular necrosis to the humeral head.
 In children the commonest injury pattern is a greenstick fracture through the
surgical neck.
 Impacted fractures of the surgical neck are usually managed with a collar and
cuff for 3 weeks followed by physiotherapy.
 More significant displaced fractures may require open reduction and fixation
or use of an intramedullary device.

Types of shoulder dislocation

 Glenohumeral dislocation (commonest): anterior shoulder dislocation most


common
 Acromioclavicular dislocation (12%): clavicle loses all attachment with the
scapula
 Sternoclavicular dislocation (uncommon)
Types of glenohumeral dislocation:
Anterior shoulder  External rotation and abduction
dislocation  35-40% recurrent (it is the commonest disorder)
 Assocociated with greater tuberosity fracture, Bankart
lesion, Hill-Sachs defect

Inferior shoulder Luxatio erecta


dislocation
Posterior shoulder  Proportion misdiagnosed.
dislocation  Rim's sign, light bulb sign.
 Assocociated with Trough sign

Superior shoulder Rare and usually follow major trauma.


dislocation

Treatment
Prompt reduction is the mainstay of treatment and is usually performed in the
emergency department. Neurovascular status must be checked pre and post reduction
and x-rays should be performed again post reduction to ensure no fracture has
occurred. In recurrent anterior dislocation there is usually a Bankart lesion and this
may be repaired surgically. Recurrent posterior dislocations may be repaired in a
similar manner to anterior lesions but using a posterior (or arthroscopic) approach.
Which of the following statements relating to menisceal tears is false?

A. The medial meniscus is most often affected

B. True locking of the knee joint may occur

C. Most established tears will heal with conservative management

D. In the chronic setting there is typically little to find on examination if


the knee is not locked

E. An arthroscopic approach may be used to treat most lesions

Menisci have no nerve or blood supply and thus heal poorly. Established tears with
associated symptoms are best managed by arthroscopic menisectomy.

Knee injury

Types of injury

Ruptured anterior  Sport injury


cruciate ligament  Mechanism: high twisting force applied to a bent knee
 Typically presents with: loud crack, pain and RAPID
swelling knee (haemoarthrosis)
 Poor healing
 Management: intense physiotherapy or surgery

Ruptured posterior  Mechanism: hyperextension injuries


cruciate ligament  Tibia lies back on the femur
 Paradoxical anterior draw test

Rupture of medial  Mechanism: leg forced into valgus via force outside the
collateral ligament leg
 Knee unstable when put into valgus position

Menisceal tear  Rotational sporting injuries


 Delayed knee swelling
 Joint locking (Patient may develop skills to "unlock"
the knee
 Recurrent episodes of pain and effusions are common,
often following minor trauma

Chondromalacia  Teenage girls, following an injury to knee e.g.


patellae Dislocation patella
 Typical history of pain on going downstairs or at rest
 Tenderness, quadriceps wasting

Dislocation of the  Most commonly occurs as a traumatic primary event,


patella either through direct trauma or through severe
contraction of quadriceps with knee streched in valgus
and external rotation
 Genu valgum, tibial torsion and high riding patella are
risk factors
 Skyline x-ray views of patella are required, although
displaced patella may be clinically obvious
 An osteochondral fracture is present in 5%
 The condition has a 20% recurrence rate

Fractured patella  2 types:

i. Direct blow to patella causing undisplaced fragments


ii. Avulsion fracture
Tibial plateau  Occur in the elderly (or following significant trauma in
fracture young)
 Mechanism: knee forced into valgus or varus, but the
knee fractures before the ligaments rupture
 Varus injury affects medial plateau and if valgus
injury, lateral plateau depressed fracture occurs
 Classified using the Schatzker system (see below)

Schatzker Classification system for tibial plateau fractures


Type Features
1- vertical split of lateral Fracture through dense bone, usually in the young. It may
condyle be virtually undisplaced, or the condylar fragment may be
pushed inferiorly and tilted
2- a vertical split of the The wedge fragement (which may be of variable size), is
lateral condyle combined displaced laterally; the joint is widened. Untreated, a
with an adjacent valgus deformity may develop
loadbearing part of the
condyle
3- depression of the The split does not extend to the edge of the plateau.
articular surface with intact
Depressed fragments may be firmly embedded in
condylar rim subchondral bone, the joint is stable
4- fragment of the medial Two injuries are seen in this category; (1) a depressed
tibial condyle fracture of osteoporotic bone in the elderly. (2) a high
energy fracture resulting in a condylar split that runs from
the intercondylar eminence to the medial cortex.
Associated ligamentous injury may be severe
5-fracture of both condyles Both condyles fractured but the column of the metaphysis
remains in continuity with the tibial shaft
6-combined condylar and High energy fracture with marked comminution
subcondylar fractures

Theme: Developmental bone disorders

A. Rickets
B. Craniocleidodysostosis
C. Achondroplasia
D. Scurvy
E. Pagets disease
F. Multiple myeloma
G. Osteogenesis imperfecta
H. Osteomalacia
I. Osteopetrosis
J. None of the above

Please select the most likely disease process to account for the clinical scenario. Each
option may be used once, more than once or not at all

9. A 15 year-old boy presents to the out-patient clinic with tiredness, recurrent


throat and chest infections, and gradual loss of vision. Multiple x-rays show
brittle bones with no differentiation between the cortex and the medulla.

You answered Rickets

The correct answer is Osteopetrosis


Osteopetrosis is an autosomal recessive condition. It is commonest in young
adults. They may present with symptoms of anaemia or thrombocytopaenia
due to decreased marrow space. Radiology reveals a lack of differentiation
between the cortex and the medulla described as marble bone. These bones are
very dense and brittle.

10. A 12 year-old boy who is small for his age presents to the clinic with poor
muscular development and hyper-mobile fingers. His x rays show multiple
fractures of the long bones and irregular patches of ossification.

You answered Craniocleidodysostosis

The correct answer is Osteogenesis imperfecta

Osteogenesis imperfecta is caused by defective osteoid formation due to


congenital inability to produce adequate intercellular substances like osteoid,
collagen and dentine. There is a failure of maturation of collagen in all the
connective tissues.Radiology may show translucent bones, multiple fractures,
particularly of the long bones, wormian bones (irregular patches of
ossification) and a trefoil pelvis.

11. A 1 year-old is brought to the Emergency Department with a history of failure


to thrive. On examination, the child is small for age and has a large head. X-
ray shows a cupped appearance of the epiphysis of the wrist.

Rickets

Rickets is the childhood form of osteomalacia. It is due to the failure of the


osteoid to ossify due to vitamin D deficiency. Symptoms start about the age of
one. The child is small for age and there is a history of failure to thrive. Bony
deformities include bowing of the femur and tibia, a large head, deformity of
the chest wall with thickening of the costochondral junction (ricketty rosary),
and a transverse sulcus in the chest caused by the pull of the diaphragm
(Harrison's sulcus). X- Rays show widening and cupping of the epiphysis of
the long bones, most readily apparent in the wrist.

Paediatric fractures

Paediatric fracture types


Type Injury pattern
Complete fracture Both sides of cortex are breached
Toddlers fracture Oblique tibial fracture in infants
Plastic deformity Stress on bone resulting in deformity without cortical disruption
Greenstick fracture Unilateral cortical breach only
Buckle fracture Incomplete cortical disruption resulting in periosteal haematoma
only

Growth plate fractures


In paediatric practice fractures may also involve the growth plate and these injuries
are classified according to the Salter- Harris system (given below):

Type Injury pattern


I Fracture through the physis only (x-ray often normal)
II Fracture through the physis and metaphysis
III Fracture through the physis and epiphyisis to include the joint
IV Fracture involving the physis, metaphysis and epiphysis
V Crush injury involving the physis (x-ray may resemble type I, and appear
normal)

As a general rule it is safer to assume that growth plate tenderness is indicative of an


underlying fracture even if the x-ray appears normal. Injuries of Types III, IV and V
will usually require surgery. Type V injuries are often associated with disruption to
growth.

Non accidental injury

 Delayed presentation
 Delay in attaining milestones
 Lack of concordance between proposed and actual mechanism of injury
 Multiple injuries
 Injuries at sites not commonly exposed to trauma
 Children on the at risk register

Pathological fractures
Genetic conditions, such as osteogenesis imperfecta, may cause pathological
fractures.

Osteogenesis imperfecta

 Defective osteoid formation due to congenital inability to produce adequate


intercellular substances like osteoid, collagen and dentine.
 Failure of maturation of collagen in all the connective tissues.
 Radiology may show translucent bones, multiple fractures, particularly of the
long bones, wormian bones (irregular patches of ossification) and a trefoil
pelvis.

Subtypes

 Type I The collagen is normal quality but insufficient quantity.


 Type II- Poor collagen quantity and quality.
 Type III- Collagen poorly formed. Normal quantity.
 Type IV- Sufficient collagen quantity but poor quality.

Osteopetrosis

 Bones become harder and more dense.


 Autosomal recessive condition.
 It is commonest in young adults.
 Radiology reveals a lack of differentiation between the cortex and the medulla
described as marble bone.

Theme: Hip fractures

A. Conservative management
B. Percutaneous pinning
C. Fracture reduction and internal fixation
D. Hemiarthroplasty
E. Total hip replacement
F. Sliding hip screw
G. Intramedullary device

For each scenario please select the most appropriate management option. Each option
may be used once, more than once or not at all.

12. A 60 year old male is admitted to A&E with a fall. He lives with his wife and
still works as a restaurant manager. He has a past history of benign prostatic
hypertrophy and is currently taking tamsulosin. He is otherwise fit and healthy.
On examination there is right hip tenderness on movement in all directions. A
hip x-ray confirms an undisplaced intracapsular fracture.

You answered Sliding hip screw

The correct answer is Percutaneous pinning

As this is undisplaced the blood supply to the femoral head may be intact and
the fracture may heal. Therefore an attempt at percutaneous fixation is
reasonable.

13. An 86 year old retired pharmacist is admitted to A&E following a fall. She
complains of right hip pain. She is known to have hypertension and is
currently on bendrofluazide. She lives alone and mobilises with a Zimmer
frame. Her right leg is shortened and externally rotated. A hip x-ray confirms a
displaced intracapsular fracture.
Hemiarthroplasty

Hemiarthroplasty is offered to older, less mobile individuals compared to


fracture reduction and fixation in younger patients.

14. A 74 year old male is admitted to A&E with a fall. He is known to have
rheumatoid arthritis and is on methotrexate and paracetamol. He lives alone in
a bungalow and enjoys playing golf. He is independent with his ADLs. He
complains of left groin pain, therefore has a hip x-ray which confirms a
displaced intracapsular fracture.

You answered Hemiarthroplasty

The correct answer is Total hip replacement

This patient has pre-existing joint disease, good level of activity and a
relatively high life expectancy, therefore THR is preferable to
hemiarthroplasty.

Hip fractures

The hip is a common site of fracture especially in osteoporotic, elderly females. The
blood supply to the femoral head runs up the neck and thus avascular necrosis is a risk
in displaced fractures.

Classification
The Garden system is one classification system in common use.

 Type I: Stable fracture with impaction in valgus.


 Type II: Complete fracture but undisplaced.
 Type III: Displaced fracture, usually rotated and angulated, but still has bony
contact.
 Type IV: Complete bony disruption.

Blood supply disruption is most common following Types III and IV.

Management of hip fractures in older adults


SIGN Guidelines
Fracture type Patient co-morbidities Management
Undisplaced Nil Internal fixation (especially if
intracapsular fracture young)
Undisplaced Major illness or Hemiarthroplasty
intracapsular fracture advanced organ specific
disease
Displaced intracapsular Nil If age <70 then internal fixation (if
fracture possible), hip arthroplasty if not
Displaced intracapsular Nil Age >70 total hip arthroplasty
fracture
Displaced intracapsular Major/ immobile Hemiarthroplasty
fracture
Extracapsular fracture Only major co- Dynamic hip screw
(non special type) morbidities affect
management
Extracapsular fracture Only major co- Usually intramedullary device
(reverse oblique, morbidities affect
transverse or sub management
trochanteric)

A typical image of an intracapsular fracture occurring in an elderly osteoporotic lady

Image sourced from Wikipedia

References
www.sign.ac.uk/guidelines/fulltext/111/index.html
Of the list below, which is not a cause of avascular necrosis?

A. Steroids

B. Sickle cell disease

C. Radiotherapy

D. Myeloma
E. Caisson disease
Causes of avascular
necrosis
P ancreatitis
L upus
A lcohol
S teroids
T rauma
I diopathic, infection
C aisson disease, collagen
vascular disease
R adiation, rheumatoid
arthritis
A myloid
G aucher disease
S ickle cell disease

Steroid containing therapy for myeloma may induce avascular necrosis, however the
disease itself does not cause it. Caisson disease as may occur in deep sea divers is a
recognised cause.

Avascular necrosis

 Cellular death of bone components due to interruption of the blood supply,


causing bone destruction
 Main joints affected are hip, scaphoid, lunate and the talus.
 It is not the same as non union. The fracture has usually united.
 Radiological evidence is slow to appear.
 Vascular ingrowth into the affected bone may occur. However, many joints
will develop secondary osteoarthritis.

Causes
P ancreatitis
L upus
A lcohol
S teroids
T rauma
I diopathic, infection
C aisson disease, collagen vascular disease
R adiation, rheumatoid arthritis
A myloid
G aucher disease
S ickle cell disease

Presentation
Usually pain. Often despite apparent fracture union.
Investigation
MRI scanning will show changes earlier than plain films.

Treatment
In fractures at high risk sites anticipation is key. Early prompt and accurate reduction
is essential.

Non weight bearing may help to facilitate vascular regeneration.

Joint replacement may be necessary, or even the preferred option (e.g. Hip in the
elderly).
Which of the following is the first radiological change likely to be apparent in a plain
radiograph of a 12 year old presenting with suspected Perthes disease

A. Multiple bone cysts

B. Sclerosis of the femoral head

C. Loss of bone density

D. Joint space narrowing

E. Collapse of the femoral head

In Catterall stage I disease there may be no radiological abnormality at all. In Stage II


disease there may be sclerosis of the femoral head.

Indication for treatment (aide memoire):Half a dozen, half a head


Those aged greater than 6 years with >50% involvement of the femoral head should
almost always be treated.

Perthes disease

Perthes disease

 Idiopathic avascular necrosis of the femoral epiphysis of the femoral head


 Impaired blood supply to femoral head, causing bone infarction. New vessels
develop and ossification occurs. The bone either heals or a subchondral
fracture occurs.

Clinical features

 Males 4x's greater than females


 Age between 2-12 years (the younger the age of onset, the better the
prognosis)
 Limp
 Hip pain
 Bilateral in 20%

Diagnosis
Plain x-ray, Technetium bone scan or magnetic resonance imaging if normal x-ray
and symptoms persist.

Catterall staging
Stage Features
Stage 1 Clinical and histological features only
Stage 2 Sclerosis with or without cystic changes and preservation of the articular
surface
Stage 3 Loss of structural integrity of the femoral head
Stage 4 Loss of acetabular integrity

Management

 To keep the femoral head within the acetabulum: cast, braces


 If less than 6 years: observation
 Older: surgical management with moderate results
 Operate on severe deformities

Prognosis
Most cases will resolve with conservative management. Early diagnosis improves
outcomes.
Theme: Upper limb injuries

A. Pulled elbow
B. Fracture of the coronoid process
C. Scaphoid fracture
D. Moteggia fracture
E. Bennets fracture
F. Fracture of the shaft of the radius and ulnar
G. Galeazzi fracture
H. Fracture of the olecranon
I. Fracture of the radial head

Please select the most likely injury for the scenario given. Each option may be used
once, more than once or not at all.

17. A 32 year old man presents with a painful swelling over the volar aspect of his
hand after receiving a hard blow to his palm. On examination, he experiences
pain on moving the wrist and on longitudinal compression of the thumb.

You answered Bennets fracture


The correct answer is Scaphoid fracture

Scaphoid fractures usually occur as a result of direct hard blow to the palm or
following a fall on the out-stretched hand. The main physical signs are
swelling and tenderness in the anatomical snuff box, and pain on wrist
movements and on longitudinal compression of the thumb

18. A 26 year old man presents to the emergency department with a swelling over
his left elbow after a fall on an outstretched hand. On examination, he has
tenderness over the proximal part of his forearm, and has severely restricted
supination and pronation movements.

You answered Moteggia fracture

The correct answer is Fracture of the radial head

Fracture of the radial head is common in young adults. It is usually caused by a


fall on the outstretched hand. On examination, there is marked local tenderness
over the head of the radius, impaired movements at the elbow, and a sharp pain
at the lateral side of the elbow at the extremes of rotation (pronation and
supination).

19. A 56 year old lady presents with a painful swelling over the lower end of the
forearm following a fall. Imaging reveals a distal radial fracture with
disruption of the distal radio-ulnar joint.

Galeazzi fracture

Galeazzi fractures occur after a fall on the hand with a rotational force
superimposed on it. On examination, there is bruising, swelling and tenderness
over the lower end of the forearm. X- Rays reveal a displaced fracture of the
radius and a prominent ulnar head due to dislocation of the inferior radio-ulnar
joint.

Upper limb fractures

Colles' fracture

 Fall onto extended outstretched hands


 Described as a dinner fork type deformity
 Classical Colles' fractures have the following 3 features:
Features of the injury
1. Transverse fracture of the radius
2. 1 inch proximal to the radio-carpal joint
3. Dorsal displacement and angulation

Smith's fracture (reverse Colles' fracture)

 Volar angulation of distal radius fragment (Garden spade deformity)


 Caused by falling backwards onto the palm of an outstretched hand or falling
with wrists flexed

Bennett's fracture

 Intra-articular fracture of the first carpometacarpal joint


 Impact on flexed metacarpal, caused by fist fights
 X-ray: triangular fragment at ulnar base of metacarpal

Monteggia's fracture

 Dislocation of the proximal radioulnar joint in association with an ulna


fracture
 Fall on outstretched hand with forced pronation
 Needs prompt diagnosis to avoid disability

Galeazzi fracture

 Radial shaft fracture with associated dislocation of the distal radioulnar joint
 Occur after a fall on the hand with a rotational force superimposed on it.
 On examination, there is bruising, swelling and tenderness over the lower end
of the forearm.
 X Rays reveal the displaced fracture of the radius and a prominent ulnar head
due to dislocation of the inferior radio-ulnar joint.

Barton's fracture

 Distal radius fracture (Colles'/Smith's) with associated radiocarpal dislocation


 Fall onto extended and pronated wrist

Scaphoid fractures

 Scaphoid fractures are the commonest carpal fractures.


 Surface of scaphoid is covered by articular cartilage with small area available
for blood vessels (fracture risks blood supply)
 Forms floor of anatomical snuffbox
 Risk of fracture associated with fall onto outstretched hand (tubercle, waist, or
proximal 1/3)
 The main physical signs are swelling and tenderness in the anatomical snuff
box, and pain on wrist movements and on longitudinal compression of the
thumb.
 Ulnar deviation AP needed for visualization of scaphoid
 Immobilization of scaphoid fractures difficult

Radial head fracture

 Fracture of the radial head is common in young adults.


 It is usually caused by a fall on the outstretched hand.
 On examination, there is marked local tenderness over the head of the radius,
impaired movements at the elbow, and a sharp pain at the lateral side of the
elbow at the extremes of rotation (pronation and supination).

Theme: Hand injuries

A. Admission and surgical debridement


B. Application of futura splint and fracture clinic review
C. Application of tubigrip bandage and fracture clinic review
D. Admission for open reduction and fixation
E. Discharge with reassurance
F. Commence oral prednisolone
G. Commence oral diclofenac

Which of the following options is the best management plan? Each option may be
used once, more than once or not at all.

20. A 42 year old skier falls and impacts his hand on his ski pole. On examination
he is tender in the anatomical snuffbox and on bimanual palpation. Xrays with
scaphoid views show no evidence of fracture.

Application of futura splint and fracture clinic review

A fracture may still be present and should be immobilised until repeat imaging
can be performed.

21. A 43 year old man falls over landing on his left hand. Although there was
anatomical snuffbox tenderness no x-rays either at the time or subsequently
have shown evidence of scaphoid fracture. He has been immobilised in a
futura splint for two weeks and is now asymptomatic.
Discharge with reassurance

This patient is at extremely low risk of having sustained a scaphoid injury and
may be discharged.

22. A builder falls from scaffolding and lands on his left hand he suffers a severe
laceration to his palm. An x-ray shows evidence of scaphoid fracture that is
minimally displaced.

You answered Application of tubigrip bandage and fracture clinic review

The correct answer is Admission and surgical debridement

This is technically an open fracture and should be debrided prior to attempted


fixation (which should occur soon after).

Scaphoid fractures:
80% of all carpal fractures
80% occur in men
80% occur at the waist of the scaphoid

Scaphoid fractures

 Scaphoid fractures are the commonest carpal fractures.


 Surface of scaphoid is covered by articular cartilage with small area available
for blood vessels (fracture risks blood supply)
 Forms floor of anatomical snuffbox
 Risk of fracture associated with fall onto outstretched hand (tubercle, waist, or
proximal third)
 Ulnar deviation AP needed for visualization of scaphoid
 Immobilization of scaphoid fractures difficult

Management
Non-displaced fractures - Casts or splints
- Percutaneous scaphoid fixation
Displaced fracture Surgical fixation, usually with a screw

Complications

 Non union of scaphoid


 Avascular necrosis of the scaphoid
 Scapholunate disruption and wrist collapse
 Degenerative changes of the adjacent joint
 Theme: Paediatric orthopaedics

A. Musculoskeletal pain
B. Congenital dysplasia of the hip
C. Slipped upper femoral epiphysis
D. Transient synovitis
E. Septic arthritis
F. Perthes disease
G. Tibial fracture

Please select the most likely diagnosis for the scenario given. Each option may
be used once, more than once or not at all.

23. A 4 year boy presents with an abnormal gait. He has a history of recent viral
illness. His WCC is 11 and ESR is 30.

Transient synovitis

Viral illnesses can be associated with transient synovitis. The WCC should
ideally be > 12 and the ESR > 40 to suggest septic arthritis.

24. A 6 year old boy presents with an groin pain. He is known to be disruptive in
class. He reports that he is bullied for being short. On examination he has an
antalgic gait and pain on internal rotation of the right hip.

Perthes disease

This child is short, has hyperactivity (disruptive behaviour) and is within the
age range for Perthes disease. Hyperactivity and short stature are associated
with Perthes disease.

25. An obese 12 year old boy is referred with pain in the left knee and hip. On
examination he has an antaglic gait and limitation of internal rotation. His knee
has normal range of passive and active movement.

Slipped upper femoral epiphysis

Similar theme to September 2012 Exam


Slipped upper femoral epiphysis is commonest in obese adolescent males. The
x-ray will show displacement of the femoral epiphysis inferolaterally.
Treatment is usually with rest and non weight bearing crutches.
Beware of attributing gait disorders to
benign processes in young children without
careful clinical and radiological
assessment.


 Paediatric orthopaedics

Diagnosis Mode of presentation Treatment Radiology


Developmental Usually diagnosed in Splints and harnesses or Initially no obvious
dysplasia of theinfancy by screening traction. In later years change on plain
hip tests. May be bilateral, osteotomy and hip films and USS gives
when disease is realignment procedures best resolution until
unilateral there may be may be needed. In 3 months of age. On
leg length inequality. arthritis a joint plain films Shentons
As disease progresses replacement may be line should form a
child may limp and needed. However, this is smooth arc
then early onset best deferred if possible
arthritis. More as it will almost
common in extended certainly require
breech babies. revision
Perthes Disease Hip pain (may be Remove pressure from X-rays will show
referred to the knee) joint to allow normal flattened femoral
usually occurring development. head. Eventually in
between 5 and 12 years Physiotherapy. Usually untreated cases the
of age. Bilateral self-limiting if femoral head will
disease in 20%. diagnosed and treated fragment.
promptly.
Slipped upper Typically seen in Bed rest and non-weight X-rays will show the
femoral obese male bearing. Aim to avoid femoral head
epiphysis adolescents. Pain is avascular necrosis. If displaced and falling
often referred to the severe slippage or risk inferolaterally (like
knee. Limitation to of it occurring then a melting ice cream
internal rotation is percutaneous pinning of cone) The
usually seen. Knee the hip may be required. Southwick angle
pain is usually present gives indication of
2 months prior to hip disease severity
slipping. Bilateral in
20%.

Theme: Eponymous fractures

A. Smith's
B. Bennett's
C. Monteggia's
D. Colle's
E. Galeazzi
F. Pott's
G. Barton's

Link the most appropriate eponymously named fracture to the scenario described.
Each scenario may be used once, more than once or not at all.

26. A 28 year old man falls on the back of his hand. On x-ray the he has a
fractured distal radius demonstrating volar displacement of the fracture.

Smith's

This is a Smith fracture (reverse Colle's fracture); unlike a Colle's this is a high
velocity injury and may require surgical correction. Note that Colles fractures
are usually dorsally displaced

27. A 38 year old window cleaner falls from his ladder. He lands on his left arm
and notices an obvious injury. An x-ray and clinical examination demonstrate
that has a fracture of the proximal ulna and associated radial dislocation

Monteggia's

This constellation of injuries is referred to as a Monteggia's fracture

28. A 32 year old man falls from scaffolding and sustains an injury to his forearm.
Clinical examination and x-ray shows that he has sustained a radial fracture
with dislocation of the inferior radio-ulna joint

Galeazzi

Isolated fracture of the radius alone can occur but is rare. Always check for
associated injury

Eponymous fractures

Colles' fracture (dinner fork deformity)

 Fall onto extended outstretched hand


 Classical Colles' fractures have the following 3 features:

1. Transverse fracture of the radius


2. 1 inch proximal to the radio-carpal joint
3. Dorsal displacement and angulation
Smith's fracture (reverse Colles' fracture)

 Volar angulation of distal radius fragment (Garden spade deformity)


 Caused by falling backwards onto the palm of an outstretched hand or falling
with wrists flexed

Bennett's fracture

 Intra-articular fracture of the first carpometacarpal joint


 Impact on flexed metacarpal, caused by fist fights
 X-ray: triangular fragment at ulnar base of metacarpal

Image sourced from Wikipedia


Monteggia's fracture

 Dislocation of the proximal radioulnar joint in association with an ulna


fracture
 Fall on outstretched hand with forced pronation
 Needs prompt diagnosis to avoid disability
Image sourced from Wikipedia
Galeazzi fracture

 Radial shaft fracture with associated dislocation of the distal radioulnar joint
 Direct blow

Pott's fracture

 Bimalleolar ankle fracture


 Forced foot eversion

Barton's fracture

 Distal radius fracture (Colles'/Smith's) with associated radiocarpal dislocation


 Fall onto extended and pronated wrist

A 54-year-old man presents to the Emergency Department with a 2 day history of a


swollen, painful left knee. You aspirate the joint to avoid admission to the orthopaedic
wards. Aspirated joint fluid shows calcium pyrophosphate crystals. Which of the
following blood tests is most useful in revealing an underlying cause?

A. Transferrin saturation
B. ACTH

C. ANA

D. Serum ferritin

E. LDH

This is a typical presentation of pseudogout. An elevated transferrin saturation may


indicate haemochromatosis, a recognised cause of pseudogout.

A high ferritin level is also seen in haemochromatosis but can be raised in a variety of
infective and inflammatory processes, including pseudogout, as part of an acute phase
response.

Pseudogout

Pseudogout is a form of microcrystal synovitis caused by the deposition of calcium


pyrophosphate dihydrate in the synovium

Risk factors

 hyperparathyroidism
 hypothyroidism
 haemochromatosis
 acromegaly
 low magnesium, low phosphate
 Wilson's disease

Features

 knee, wrist and shoulders most commonly affected


 joint aspiration: weakly-positively birefringent rhomboid shaped crystals
 x-ray: chondrocalcinosis

Management

 aspiration of joint fluid, to exclude septic arthritis


 NSAIDs or intra-articular, intra-muscular or oral steroids as for gout

A 65-year-old Asian female presents with an extracapsular neck of femur fracture.


Investigations show:

Calcium 2.07 mmol/l (2.20-2.60 mmol/l)


Phosphate 0.66 mmol/l (0.8-1.40 mmol/l)
ALP 256 IU/l (44-147 IU/l)
What is the most likely diagnosis?

A. Bone tuberculosis

B. Hypoparathyroidism

C. Myeloma

D. Osteomalacia

E. Paget's disease
Osteomalacia

 low: calcium,
phosphate
 raised: alkaline
phosphatase

The low calcium and phosphate combined with the raised alkaline phosphatase point
towards osteomalacia.

Osteomalacia

Basics

 normal bony tissue but decreased mineral content


 rickets if when growing
 osteomalacia if after epiphysis fusion

Types

 vitamin D deficiency e.g. malabsorption, lack of sunlight, diet


 renal failure
 drug induced e.g. anticonvulsants
 vitamin D resistant; inherited
 liver disease, e.g. cirrhosis

Features

 rickets: knock-knee, bow leg, features of hypocalcaemia


 osteomalacia: bone pain, fractures, muscle tenderness, proximal myopathy

Investigation
 low calcium, phosphate, 25(OH) vitamin D
 raised alkaline phosphatase
 x-ray: children - cupped, ragged metaphyseal surfaces; adults - translucent
bands (Looser's zones or pseudofractures)

Treatment

 calcium with vitamin D tablets

A 78-year-old woman is discharged following a fractured neck of femur. On review


she is making good progress but consideration is given to secondary prevention of
further fractures. Unfortunately the orthogeriatricians are all on annual leave and the
consultant has asked you to arrange suitable management. Which is the best option?

A. Alendronate

B. Alendronate, calcium and vitamin D supplementation

C. Strontium

D. Arrange a DEXA scan

E. Hormone replacement therapy

A bisphosphonate, calcium and vitamin D supplementation should be given to all


patients aged over 75 years after having a fracture. A DEXA scan is only needed of
the patient is aged below 75 years. Hormone replacement therpay has been shown to
reduce vertebral and non vertebral fractures, however the risks of cardiovascular
disease and breast malignancy make this a less favourable option.

Osteoporosis: secondary prevention

NICE guidelines were updated in 2008 on the secondary prevention of osteoporotic


fractures in postmenopausal women.

Key points include

 Treatment is indicated following osteoporotic fragility fractures in


postmenopausal women who are confirmed to have osteoporosis (a T-score of
- 2.5 SD or below).
 In women aged 75 years or older, a DEXA scan may not be required 'if the
responsible clinician considers it to be clinically inappropriate or unfeasible'
 Vitamin D and calcium supplementation should be offered to all women
unless the clinician is confident they have adequate calcium intake and are
vitamin D replete
 Alendronate is first-line
 Around 25% of patients cannot tolerate alendronate, usually due to upper
gastrointestinal problems. These patients should be offered risedronate or
etidronate (see treatment criteria below)
 Strontium ranelate and raloxifene are recommended if patients cannot tolerate
bisphosphonates (see treatment criteria below)

Supplementary notes on treatment

Bisphosphonates

 Alendronate, risedronate and etidronate are all licensed for the prevention and
treatment of post-menopausal and glucocorticoid-induced osteoporosis
 All three have been shown to reduce the risk of both vertebral and non-
vertebral fractures although alendronate, risedronate may be superior to
etidronate in preventing hip fractures
 Ibandronate is a once-monthly oral bisphosphonate

Vitamin D and calcium

 Poor evidence base to suggest reduced fracture rates in the general population
at risk of osteoporotic fractures - may reduce rates in frail, housebound
patients

Raloxifene - selective oestrogen receptor modulator (SERM)

 Has been shown to prevent bone loss and to reduce the risk of vertebral
fractures, but has not yet been shown to reduce the risk of non-vertebral
fractures
 Has been shown to increase bone density in the spine and proximal femur
 May worsen menopausal symptoms
 Increased risk of thromboembolic events
 May decrease risk of breast cancer

Strontium ranelate

 'Dual action bone agent' - increases deposition of new bone by osteoblasts and
reduces the resorption of bone by osteoclasts
 Strong evidence base, may be second-line treatment in near future
 Increased risk of thromboembolic events

Which of the following statements relating to avascular necrosis is false?

A. When associated with fracture may occur despite the


radiological evidence of fracture union.

B. Pain and stiffness will typically precede radiological


evidence of the condition.

C. Drilling of affected bony fragments may be used to


facilitate angiogenesis where arthroplasty is not
warranted.

D. The earliest detectable radiological evidence is a


radiolucency of the affected area coupled with
subchondral collapse.

E. It is less likely when prompt anatomical alignment of


fracture fragments is achieved.
Avascular necrosis- radiological
changes occur late.

Radiolucency and subchondral collapse are late changes. The earliest evidence on
plain films is the affected area appearing as being more radio-opaque due to
hyperaemia and resorption of the neighboring area. It may be diagnosed earlier using
bone scans and MRI.

Avascular necrosis

 Cellular death of bone components due to interruption of the blood supply,


causing bone destruction
 Main joints affected are hip, scaphoid, lunate and the talus.
 It is not the same as non union. The fracture has usually united.
 Radiological evidence is slow to appear.
 Vascular ingrowth into the affected bone may occur. However, many joints
will develop secondary osteoarthritis.

Causes
P ancreatitis
L upus
A lcohol
S teroids
T rauma
I diopathic, infection
C aisson disease, collagen vascular disease
R adiation, rheumatoid arthritis
A myloid
G aucher disease
S ickle cell disease

Presentation
Usually pain. Often despite apparent fracture union.
Investigation
MRI scanning will show changes earlier than plain films.

Treatment
In fractures at high risk sites anticipation is key. Early prompt and accurate reduction
is essential.

Non weight bearing may help to facilitate vascular regeneration.

Joint replacement may be necessary, or even the preferred option (e.g. Hip in the
elderly).
Which of the following statements relating to avascular necrosis is false?

A. When associated with fracture may occur despite the


radiological evidence of fracture union.

B. Pain and stiffness will typically precede radiological


evidence of the condition.

C. Drilling of affected bony fragments may be used to


facilitate angiogenesis where arthroplasty is not
warranted.

D. The earliest detectable radiological evidence is a


radiolucency of the affected area coupled with
subchondral collapse.

E. It is less likely when prompt anatomical alignment of


fracture fragments is achieved.
Avascular necrosis- radiological
changes occur late.

Radiolucency and subchondral collapse are late changes. The earliest evidence on
plain films is the affected area appearing as being more radio-opaque due to
hyperaemia and resorption of the neighboring area. It may be diagnosed earlier using
bone scans and MRI.

Avascular necrosis

 Cellular death of bone components due to interruption of the blood supply,


causing bone destruction
 Main joints affected are hip, scaphoid, lunate and the talus.
 It is not the same as non union. The fracture has usually united.
 Radiological evidence is slow to appear.
 Vascular ingrowth into the affected bone may occur. However, many joints
will develop secondary osteoarthritis.
Causes
P ancreatitis
L upus
A lcohol
S teroids
T rauma
I diopathic, infection
C aisson disease, collagen vascular disease
R adiation, rheumatoid arthritis
A myloid
G aucher disease
S ickle cell disease

Presentation
Usually pain. Often despite apparent fracture union.

Investigation
MRI scanning will show changes earlier than plain films.

Treatment
In fractures at high risk sites anticipation is key. Early prompt and accurate reduction
is essential.

Non weight bearing may help to facilitate vascular regeneration.

Joint replacement may be necessary, or even the preferred option (e.g. Hip in the
elderly)
Theme: Diseases affecting the spine

A. Spondylolysis
B. Spina bifida occulta
C. Spondylolisthesis
D. Meningomyelocele
E. Meningocele
F. Scoliosis - non structural
G. Scoliosis
H. Ankylosing spondylitis
I. Scheuermann's disease

Please select the most likely underlying diagnosis for the condition described. Each
condition may be used once, more than once or not at all.

33. A 19 year old female is involved in an athletics event. She has just completed
the high jump when she suddenly develops severe back pain and weakness
affecting both her legs. on examination she has a prominent sacrum and her
lower back is painful.
Spondylolisthesis

Theme from September 2012 Exam


Young athletic females are the group most frequently affected by
spondylolythesis who have a background of spondylolysis. Whilst the latter
condition is a risk factor for spondylolythesis the former condition is most
likely in a young athletic female who presents with sudden pain.

34. A 15 year old boy is brought to the clinic by his mother who is concerned that
he has a mark overlying his lower spine. On examination the boy has a patch
of hair overlying his lower lumbar spine and a birth mark at the same location.
Lower limb neurological examination is normal.

Spina bifida occulta

Spina bifida occulta is a common condition and may affect up to 10% of the
population. The more severe types of spina bifida have more characteristic
skin changes. Occasionally the unwary surgeon is persuaded to operate on
these "cutaneous" changes and we would advocate performing an MRI scan
prior to any such surgical procedure in this region.

35. A 19 year old female presents to the clinic with progressive pain in her neck
and back. The condition has been progressively worsening over the past 6
months. She has not presented previously because she was an inpatient with a
disease flare of ulcerative colitis. On examination she has a stiff back with
limited spinal extension on bending forwards.

Ankylosing spondylitis

Ankylosing spondylitis is associated with HLA B27, there is a strong


association with ulcerative colitis in such individuals. The clinical findings are
usually of a kyphosis affecting the cervical and thoracic spine. Considerable
symptomatic benefit may be obtained using non steroidal anti inflammatory
drugs. These should be used carefully in patients with inflammatory bowel
disease who may be taking steroids.

Diseases affecting the vertebral column

Ankylosing  Chronic inflammatory disorder affecting the axial skeleton


spondylitis  Sacro-ilitis is a usually visible in plain films
 Up to 20% of those who are HLA B27 positive will
develop the condition
 Affected articulations develop bony or fibrous changes
 Typical spinal features include loss of the lumbar lordosis
and progressive kyphosis of the cervico-thoracic spine

Scheuermann's  Epiphysitis of the vertebral joints is the main pathological


disease process
 Predominantly affects adolescents
 Symptoms include back pain and stiffness
 X-ray changes include epiphyseal plate disturbance and
anterior wedging
 Clinical features include progressive kyphosis (at least 3
vertebrae must be involved)
 Minor cases may be managed with physiotherapy and
analgesia, more severe cases may require bracing or
surgical stabilisation

Scoliosis  Consists of curvature of the spine in the coronal plane


 Divisible into structural and non structural, the latter being
commonest in adolescent females who develop minor
postural changes only. Postural scoliosis will typically
disappear on manoeuvres such as bending forwards
 Structural scoliosis affects > 1 vertebral body and is
divisible into idiopathic, congential and neuromuscular in
origin. It is not correctable by alterations in posture
 Within structural scoliosis, idiopathic is the most common
type
 Severe, or progressive structural disease is often managed
surgically with bilateral rod stabilisation of the spine

Spina bifida  Non fusion of the vertebral arches during embryonic


development
 Three categories; myelomeningocele, spina bifida occulta
and meningocele
 Myelomeningocele is the most severe type with associated
neurological defects that may persist in spite of anatomical
closure of the defect
 Up to 10% of the population may have spina bifida
occulta, in this condition the skin and tissues (but not not
bones) may develop over the distal cord. The site may be
identifiable by a birth mark or hair patch
 The incidence of the condition is reduced by use of folic
acid supplements during pregnancy

Spondylolysis  Congenital or acquired deficiency of the pars


interarticularis of the neural arch of a particular vertebral
body, usually affects L4/ L5
 May be asymptomatic and affects up to 5% of the
population
 Spondylolysis is the commonest cause of spondylolisthesis
in children
 Asymptomatic cases do not require treatment

Spondylolisthesis  This occurs when one vertebra is displaced relative to its


immediate inferior vertebral body
 May occur as a result of stress fracture or spondylolysis
 Traumatic cases may show the classic "Scotty Dog"
appearance on plain films
 Treatment depends upon the extent of deformity and
associated neurological symptoms, minor cases may be
actively monitored. Individuals with radicular symptoms
or signs will usually require spinal decompression and
stabilisation

Theme: Management of fractures

A. Discharge home with arm sling and fracture clinic appointment


B. Discharge home with futura splint and fracture clinic appointment
C. Admit for open reduction and fixation
D. Fasciotomy
E. Active observation for progression of neurovascular compromise
F. Reduction of fracture in casualty and application of plaster backslab,
followed by discharge home.

Please select the most appropriate immediate management for the fracture scenarios
given. Each option may be used once, more than once or not at all.

36. A 22 year old rugby player falls onto an outstretched hand and sustains a
fracture of the distal radius. The x-ray shows a dorsally angulated comminuted
fracture.

You answered Reduction of fracture in casualty and application of plaster


backslab, followed by discharge home.

The correct answer is Admit for open reduction and fixation

Unlike an osteoporotic fracture in an elderly lady this is a high velocity injury


and will require surgical fixation.

37. A 10 year old boy undergoes a delayed open reduction and fixation of a
significantly displaced supracondylar fracture. On the ward he complains of
significant forearm pain and paraesthesia of the hand. Radial pulse is normal.
You answered Active observation for progression of neurovascular
compromise

The correct answer is Fasciotomy

The delay is the significant factor here. These injuries often have
neurovascular compromise and inactivity now places him at risk of developing
complications. In compartment syndrome the loss of arterial pulsation occurs
late.

38. A 28 year old man falls onto an outstretched hand. On examination there is
tenderness of the anatomical snuffbox. However, forearm and hand x-rays are
normal.

Discharge home with futura splint and fracture clinic appointment

This could well be a scaphoid fracture and should be temporarily immobilised


pending further review. A futura splint will immobilise better than an arm sling
for this problem.

Fracture management

 Bony injury resulting in a fracture may arise from trauma (excessive forces
applied to bone), stress related (repetitive low velocity injury) or pathological
(abnormal bone which fractures during normal use of following minimal
trauma)
 Diagnosis involves not just evaluating the fracture ; such as site and type of
injury but also other associated injuries and distal neurovascular deficits. This
may entail not just clinical examination but radiographs of proximal and distal
joints.
 When assessing x-rays it is important to assess for changes in length of the
bone, the angulation of the distal bone, rotational effects, presence of material
such as glass.

Fracture types
Fracture type Description
Oblique fracture Fracture lies obliquely to long axis of bone
Comminuted fracture >2 fragments
Segmental fracture More than one fracture along a bone
Transverse fracture Perpendicular to long axis of bone
Spiral fracture Severe oblique fracture with rotation along long axis of bone
Open Vs Closed
It is also important to distinguish open from closed injuries. The most common
classification system for open fractures is the Gustilo and Anderson classification
system (given below):

Grade Injury
1 Low energy wound <1cm
2 Greater than 1cm wound with moderate soft tissue damage
3 High energy wound > 1cm with extensive soft tissue damage
3 A (sub group of 3) Adequate soft tissue coverage
3 B (sub group of 3) Inadequate soft tissue coverage
3 C (sub group of 3) Associated arterial injury

Key points in management of fractures

 Immobilise the fracture including the proximal and distal joints


 Carefully monitor and document neurovascular status, particularly following
reduction and immobilisation
 Manage infection including tetanus prophylaxis
 IV broad spectrum antibiotics for open injuries
 As a general principle all open fractures should be thoroughly debrided ( and
internal fixation devices avoided or used with extreme caution)
 Open fractures constitute an emergency and should be debrided and lavaged
within 6 hours of injury

A 4 year old boy falls and sustains a fracture to the growth plate of his right wrist.
Which of the following systems is used to classify the injury?

A. Salter - Harris system

B. Weber system

C. Gustilo - Anderson system

D. Garden system

E. None of the above

The Salter - Harris system is most commonly used. The radiological signs in Type 1
and 5 injuries may be identical. Which is unfortunate as type 5 injuries do not do well
(and may be missed!)

Paediatric fractures

Paediatric fracture types


Type Injury pattern
Complete fracture Both sides of cortex are breached
Toddlers fracture Oblique tibial fracture in infants
Plastic deformity Stress on bone resulting in deformity without cortical disruption
Greenstick fracture Unilateral cortical breach only
Buckle fracture Incomplete cortical disruption resulting in periosteal haematoma
only

Growth plate fractures


In paediatric practice fractures may also involve the growth plate and these injuries
are classified according to the Salter- Harris system (given below):

Type Injury pattern


I Fracture through the physis only (x-ray often normal)
II Fracture through the physis and metaphysis
III Fracture through the physis and epiphyisis to include the joint
IV Fracture involving the physis, metaphysis and epiphysis
V Crush injury involving the physis (x-ray may resemble type I, and appear
normal)

As a general rule it is safer to assume that growth plate tenderness is indicative of an


underlying fracture even if the x-ray appears normal. Injuries of Types III, IV and V
will usually require surgery. Type V injuries are often associated with disruption to
growth.

Non accidental injury

 Delayed presentation
 Delay in attaining milestones
 Lack of concordance between proposed and actual mechanism of injury
 Multiple injuries
 Injuries at sites not commonly exposed to trauma
 Children on the at risk register

Pathological fractures
Genetic conditions, such as osteogenesis imperfecta, may cause pathological
fractures.

Osteogenesis imperfecta

 Defective osteoid formation due to congenital inability to produce adequate


intercellular substances like osteoid, collagen and dentine.
 Failure of maturation of collagen in all the connective tissues.
 Radiology may show translucent bones, multiple fractures, particularly of the
long bones, wormian bones (irregular patches of ossification) and a trefoil
pelvis.
Subtypes

 Type I The collagen is normal quality but insufficient quantity.


 Type II- Poor collagen quantity and quality.
 Type III- Collagen poorly formed. Normal quantity.
 Type IV- Sufficient collagen quantity but poor quality.

Osteopetrosis

 Bones become harder and more dense.


 Autosomal recessive condition.
 It is commonest in young adults.
 Radiology reveals a lack of differentiation between the cortex and the medulla
described as marble bone.

Theme: Bone disease

A. Osteogenesis imperfecta
B. Osteoporosis
C. Rickets
D. Pagets disease
E. Chondrosarcoma
F. Metastatic breast cancer

Please select the most likely diagnosis for the scenario given. Each option may be
used once, more than once or not at all.

40. A 66 year old lady presents with pain in her right hip. It has been increasing
over the previous three weeks and waking her from sleep. On examination she
is tender on internal rotation. Blood tests reveal a mildly elevated serum
calcium and alkaline phosphatase levels.

Metastatic breast cancer

Increasing pain at rest, together with increased serum calcium and alkaline
phosphatase are most likely to represent metastatic tumour to bone.
Chondrosarcomas do occur in the pelvis but are not associated with increased
serum calcium and typically have a longer history.

41. A 73 year old man presents with pain in the right leg. It is most uncomfortable
on walking. On examination he has a deformity of his right femur, which on x-
ray is thickened and sclerotic. His serum alkaline phosphatase is elevated, but
calcium is within normal limits.
You answered Osteoporosis

The correct answer is Pagets disease

This is a typical scenario for Pagets disease.

42. A 73 year old lady presents with pain in her left hip. She was walking around
the house when she tripped over a rug and fell over. Apart from temporal
arteritis which is well controlled with prednisolone she is otherwise well. On
examination he leg is shorted and externally rotated.Her serum alkaline
phosphatase and calcium are normal.

Osteoporosis

The combination of age, female gender and steroids coupled with hip pain on
minor trauma are strongly suggestive of osteoporosis.

Bone disease

Disease Features Treatment


Pagets  Focal bone resorption followed by Bisphosphonates
excessive and chaotic bone
deposition
 Affects (in order): spine, skull,
pelvis and femur
 Serum alkaline phosphatase raised
(other parameters normal)
 Abnormal thickened, sclerotic
bone on x-rays
 Risk of cardiac failure with >15%
bony involvement
 Small risk of sarcomatous change

Osteoporosis  Excessive bone resorption Bisphosphonates, calcium


resulting in demineralised bone and vitamin D
 Commoner in old age
 Increased risk of pathological
fracture, otherwise asymptomatic
 Alkaline phosphatase normal,
calcium normal

Secondary bone  Bone destruction and tumour Radiotherapy, prophylactic


tumours infiltration fixation and analgesia
 Mirel scoring used to predict risk
of fracture
 Appearances depend on primary
(e.g.sclerotic - prostate, lytic -
breast)
 Elevated serum calcium and
alkaline phosphatase may be seen

Theme: Shoulder pain

A. Impingement syndrome
B. Rotator cuff tear
C. Adhesive capsulitis
D. Calcific tendonitis
E. Biceps tendon rupture
F. Parsonage - Turner syndrome
G. Labral tear

Please select the most likely cause for shoulder pain from the list. Each option may be
used once, more than once or not at all.

43. A 63 year old lady undergoes an axillary clearance for breast cancer. She
makes steady progress. However, 8 weeks post operatively she still suffers
from severe shoulder pain. On examination she has reduced active movements
in all planes and loss of passive external rotation.

You answered Calcific tendonitis

The correct answer is Adhesive capsulitis

Frozen shoulder passes through an initial painful stage followed by a period of


joint stiffness. With physiotherapy the problem will usually resolve although it
may take up to 2 years to do so.

44. A 78 year old man complains of a long history of shoulder pain and more
recently weakness. On examination active attempts at abduction are impaired.
Passive movements are normal.

Rotator cuff tear

Rotator cuff tears are common in elderly people and may occur following
minor trauma or as a result of long standing impingement. Tears greater than
2cm should generally be repaired surgically.
45. A 28 year old man complains of pain and weakness in the shoulder. He has
recently been unwell with glandular fever from which he is fully recovered. On
examination there is some evidence of muscle wasting and a degree of winging
of the scapula. Power during active movements is impaired.

You answered Impingement syndrome

The correct answer is Parsonage - Turner syndrome

This is a peripheral neuropathy that may complicate viral illnesses and usually
resolves spontaneously.

Deep seated pain in the proximal


forearm especially during the night and
at rest may be due to tumour, especially
metastatic lesions.

Shoulder disorders

Proximal humerus fractures

 Very common. Usually through the surgical neck. Number of classification


systems though for practical purposes describing the number of fracture
fragments is probably easier. Some key points:
 It is rare to have fractures through the anatomical neck.
 Anatomical neck fractures which are displaced by >1cm carry a risk of
avascular necrosis to the humeral head.
 In children the commonest injury pattern is a greenstick fracture through the
surgical neck.
 Impacted fractures of the surgical neck are usually managed with a collar and
cuff for 3 weeks followed by physiotherapy.
 More significant displaced fractures may require open reduction and fixation
or use of an intramedullary device.

Types of shoulder dislocation

 Glenohumeral dislocation (commonest): anterior shoulder dislocation most


common
 Acromioclavicular dislocation (12%): clavicle loses all attachment with the
scapula
 Sternoclavicular dislocation (uncommon)
Types of glenohumeral dislocation:
Anterior shoulder  External rotation and abduction
dislocation  35-40% recurrent (it is the commonest disorder)
 Assocociated with greater tuberosity fracture, Bankart
lesion, Hill-Sachs defect

Inferior shoulder Luxatio erecta


dislocation
Posterior shoulder  Proportion misdiagnosed.
dislocation  Rim's sign, light bulb sign.
 Assocociated with Trough sign

Superior shoulder Rare and usually follow major trauma.


dislocation

Treatment
Prompt reduction is the mainstay of treatment and is usually performed in the
emergency department. Neurovascular status must be checked pre and post reduction
and x-rays should be performed again post reduction to ensure no fracture has
occurred. In recurrent anterior dislocation there is usually a Bankart lesion and this
may be repaired surgically. Recurrent posterior dislocations may be repaired in a
similar manner to anterior lesions but using a posterior (or arthroscopic) approach.
Theme: Knee injuries

A. Anterior cruciate ligament rupture


B. Posterior cruciate ligament rupture
C. Medial collateral ligament tear
D. Lateral collateral ligament tear
E. Torn meniscus
F. Chondromalacia patellae
G. Dislocated patella
H. Fractured patella
I. Tibial plateau fracture

What is the most likely injury for scenario given? Each option may be used once,
more than once or not at all.

46. A 38 year old man is playing football when he slips over during a tackle. His
knee is painful immediately following the fall. Several hours later he notices
that the knee has become swollen. Following a course of non steroidal anti
inflammatory drugs and rest the situation improves. However, complains of
recurrent pain. On assessment in clinic you notice that it is impossible to fully
extend the knee, although the patient is able to do so when asked.

Torn meniscus
Theme from September 2012 Exam
Twisting sporting injuries followed by delayed onset of knee swelling and
locking are strongly suggestive of a menisceal tear. Arthroscopic menisectomy
is the usual treatment.

47. A 34 year old woman is a passenger in a car during an accident. Her knee hits
the dashboard. On examination the tibia looks posterior compared to the non
injured knee.

Posterior cruciate ligament rupture

In ruptured posterior cruciate ligament the tibia lies back on the femur and can
be drawn forward during a paradoxical draw test.

48. A 28 year old professional footballer is admitted to the emergency department.


During a tackle he is twisted with his knee flexed. He hears a loud crack and
his knee rapidly becomes swollen.

Anterior cruciate ligament rupture

This is common in footballers as the football boot studs stick to the ground and
high twisting force is applied to a flexed knee. Rapid joint swelling also
supports the diagnosis.

Knee injury

Types of injury

Ruptured anterior  Sport injury


cruciate ligament  Mechanism: high twisting force applied to a bent knee
 Typically presents with: loud crack, pain and RAPID
swelling knee (haemoarthrosis)
 Poor healing
 Management: intense physiotherapy or surgery

Ruptured posterior  Mechanism: hyperextension injuries


cruciate ligament  Tibia lies back on the femur
 Paradoxical anterior draw test

Rupture of medial  Mechanism: leg forced into valgus via force outside the
collateral ligament leg
 Knee unstable when put into valgus position

Menisceal tear  Rotational sporting injuries


 Delayed knee swelling
 Joint locking (Patient may develop skills to "unlock"
the knee
 Recurrent episodes of pain and effusions are common,
often following minor trauma

Chondromalacia  Teenage girls, following an injury to knee e.g.


patellae Dislocation patella
 Typical history of pain on going downstairs or at rest
 Tenderness, quadriceps wasting

Dislocation of the  Most commonly occurs as a traumatic primary event,


patella either through direct trauma or through severe
contraction of quadriceps with knee streched in valgus
and external rotation
 Genu valgum, tibial torsion and high riding patella are
risk factors
 Skyline x-ray views of patella are required, although
displaced patella may be clinically obvious
 An osteochondral fracture is present in 5%
 The condition has a 20% recurrence rate

Fractured patella  2 types:

i. Direct blow to patella causing undisplaced fragments


ii. Avulsion fracture
Tibial plateau  Occur in the elderly (or following significant trauma in
fracture young)
 Mechanism: knee forced into valgus or varus, but the
knee fractures before the ligaments rupture
 Varus injury affects medial plateau and if valgus
injury, lateral plateau depressed fracture occurs
 Classified using the Schatzker system (see below)

Schatzker Classification system for tibial plateau fractures


Type Features
1- vertical split of lateral Fracture through dense bone, usually in the young. It may
condyle be virtually undisplaced, or the condylar fragment may be
pushed inferiorly and tilted
2- a vertical split of the The wedge fragement (which may be of variable size), is
lateral condyle combined displaced laterally; the joint is widened. Untreated, a
with an adjacent valgus deformity may develop
loadbearing part of the
condyle
3- depression of the The split does not extend to the edge of the plateau.
articular surface with intactDepressed fragments may be firmly embedded in
condylar rim subchondral bone, the joint is stable
4- fragment of the medial Two injuries are seen in this category; (1) a depressed
tibial condyle fracture of osteoporotic bone in the elderly. (2) a high
energy fracture resulting in a condylar split that runs from
the intercondylar eminence to the medial cortex.
Associated ligamentous injury may be severe
5-fracture of both condyles Both condyles fractured but the column of the metaphysis
remains in continuity with the tibial shaft
6-combined condylar and High energy fracture with marked comminution
subcondylar fractures
A 10 year old boy is referred to the orthopaedic clinic with symptoms of right knee
pain. He has suffered pain for the past 3 months and the pain typically lasts for several
hours. On examination he walks with an antalgic gait and has apparent right leg
shortening. The right knee is normal but the right hip reveals pain on internal and
external rotation. Imaging shows flattening of the femoral head. Which of the
following is the most likely underlying diagnosis?

A. Osteogenesis imperfecta

B. Child abuse

C. Osteosarcoma

D. Osteopetrosis

E. Perthes disease

This is a typical description of Perthes disease. Management involves keeping the


femoral head in the acetabulum by braces, casts or surgery.

Perthes disease

Perthes disease

 Idiopathic avascular necrosis of the femoral epiphysis of the femoral head


 Impaired blood supply to femoral head, causing bone infarction. New vessels
develop and ossification occurs. The bone either heals or a subchondral
fracture occurs.

Clinical features

 Males 4x's greater than females


 Age between 2-12 years (the younger the age of onset, the better the
prognosis)
 Limp
 Hip pain
 Bilateral in 20%

Diagnosis
Plain x-ray, Technetium bone scan or magnetic resonance imaging if normal x-ray
and symptoms persist.

Catterall staging
Stage Features
Stage 1 Clinical and histological features only
Stage 2 Sclerosis with or without cystic changes and preservation of the articular
surface
Stage 3 Loss of structural integrity of the femoral head
Stage 4 Loss of acetabular integrity

Management

 To keep the femoral head within the acetabulum: cast, braces


 If less than 6 years: observation
 Older: surgical management with moderate results
 Operate on severe deformities

Prognosis
Most cases will resolve with conservative management. Early diagnosis improves
outcomes.
Which statement relating to talipes equinovarus is untrue?

A. It has an annual incidence of around 1 in 1000 in the UK.

B. The muscles involved in the disorder are intrinsically abnormal.

C. The cuboid is classically displaced medially.

D. All cases should be treated with an Ilizarov frame initially unless


there is minor deformity.

E. The talocalcaneal angle is typically less than 20 degrees in club foot.

In most cases of Club Foot conservative measures should be tried first. The Ponsetti
method is a popular approach. Severe cases may benefit from Ilizarov frame re-
aligment.

Talipes Equinovarus

Congenital talipes equinovarus.


Features:
 Equinus of the hindfoot.
 Adduction and varus of the midfoot.
 High arch.

Most cases in developing countries. Incidence in UK is 1 per 1000 live births. It is


more common in males and is bilateral in 50% cases. There is a strong familial
link(1). It may also be associated with other developmental disorders such as Down's
syndrome.

Key anatomical deformities (2):

 Adducted and inverted calcaneus


 Wedge shaped distal calcaneal articular surface
 Severe Tibio-talar plantar flexion.
 Medial Talar neck inclination
 Displacement of the navicular bone (medially)
 Wedge shaped head of talus
 Displacement of the cuboid (medially)

Management
Conservative first, the Ponseti method is best described and gives comparable results
to surgery. It consists of serial casting to mold the foot into correct shape. Following
casting around 90% will require a Achilles tenotomy. This is then followed by a phase
of walking braces to maintain the correction.

Surgical correction is reserved for those cases that fail to respond to conservative
measures. The procedures involve multiple tenotomies and lengthening procedures. In
patients who fail to respond surgically an Ilizarov frame reconstruction may be
attempted and gives good results.

References
1. Wynne-Davies R, Littlejohn A, Gormley J. Aetiology and interrelationship of some
common skeletal deformities. (Talipes equinovarus and calcaneovalgus, metatarsus
varus, congenital dislocation of the hip, and infantile idiopathic scoliosis). J Med
Genet. 1982 Oct;19(5):321-8.
2. Horn BD, Davidson RS. Current treatment of clubfoot in infancy and childhood.
Foot Ankle Clin. 2010 Jun;15(2):235-43.
3. Clarke NM, Uglow MG, Valentine KM. Comparison of Ponseti Versus Surgical
Treatment in Congenital Talipes Equinovarus. J Foot Ankle Surg. 2011 Jun 14.
Which of the following is least likely to impair bone fracture healing?

A. Radiotherapy

B. Osteoporosis

C. Administration of non steroidal anti inflammatory drugs


D. Preservation of periosteum

E. Presence of osteomyelitic sequestra

Periosteal preservation helps fractures to heal.

Fracture healing

Bone fracture
- Bleeding vessels in the bone and periosteum
- Clot and haematoma formation
- The clot organises over a week (improved structure and collagen)
- The periosteum contains osteoblasts which produce new bone
- Mesenchymal cells produce cartilage (fibrocartilage and hyaline cartilage) in the soft
tissue around the fracture
- Connective tissue + hyaline cartilage = callus
- As the new bone approaches the new cartilage, endochondral ossification occurs to
bridge the gap
- Trabecular bone forms
- Trabecular bone is resorbed by osteoclasts and replaced with compact bone

Factors Affecting Fracture Healing

 Age
 Malnutrition
 Bone disorders: osteoporosis
 Systemic disorders: diabetes, Marfan's syndrome and Ehlers-Danlos syndrome
cause abnormal musculoskeletal healing.
 Drugs: steroids, non steroidal anti inflammatory agents.
 Type of bone: Cancellous (spongy) bone fractures are usually more stable,
involve greater surface areas, and have a better blood supply than cortical
(compact) bone fractures.
 Degree of Trauma: The more extensive the injury to bone and surrounding soft
tissue, the poorer the outcome.
 Vascular Injury: Especially the femoral head, talus, and scaphoid bones.
 Degree of Immobilization
 Intra-articular Fractures: These fractures communicate with synovial fluid,
which contains collagenases that retard bone healing.
 Separation of Bone Ends: Normal apposition of fracture fragments is needed
for union to occur. Inadequate reduction, excessive traction, or interposition of
soft tissue will prevent healing.
 Infection
 Theme: Disorders of the hip

A. Perthes disease
B. Developmental dysplasia of the hip
C. Osteoarthritis
D. Slipped upper femoral epiphysis
E. Septic arthritis
F. Rheumatoid arthritis
G. Intra capsular fracture of the femoral neck
H. Extra capsular fracture of the femoral neck

Please select the most likely diagnosis for the scenario given. Each option may
be used once, more than once or not at all.

52. An obese 14 year old boy presents with difficulty running and mild knee and
hip pain. There is no antecedent history of trauma. On examination internal
rotation is restricted but the knee is normal with full range of passive
movement possible and no evidence of effusions. Both the C-reactive protein
and white cell count are normal.

Slipped upper femoral epiphysis

Slipped upper femoral epiphysis is the commonest adolescent hip disorder. It


occurs most commonly in obese males. It may often present as knee pain
which is usually referred from the ipsilateral hip. The knee itself is normal.
The hip often limits internal rotation. The diagnosis is easily missed. X-rays
will show displacement of the femoral epiphysis and the degree of its
displacement may be calculated using the Southwick angle. Treatment is
directed at preventing further slippage which may result in avascular necrosis
of the femoral head.

53. A 6 year old boy presents with pain in the hip it is present on activity and has
been worsening over the past few weeks. There is no history of trauma. He
was born by normal vaginal delivery at 38 weeks gestation On examination he
has an antalgic gait and limitation of active and passive movement of the hip
joint in all directions. C-reactive protein is mildly elevated at 10 but the white
cell count is normal.

Perthes disease

This is a typical presentation for Perthes disease. X-ray may show flattening of
the femoral head or fragmentation in more advanced cases.

54. A 30 year old man presents with severe pain in the left hip it has been present
on and off for many years. He was born at 39 weeks gestation by emergency
caesarean section after a long obstructed breech delivery. He was slow to walk
and as a child was noted to have an antalgic gait. He was a frequent attender at
the primary care centre and the pains dismissed as growing pains. X-rays show
almost complete destruction of the femoral head and a narrow acetabulum.
Developmental dysplasia of the hip

Developmental dysplasia of the hip. Usually diagnosed by Barlow and


Ortolani tests in early childhood. Most Breech deliveries are also routinely
subjected to USS of the hip joint. At this young age an arthrodesis may be
preferable to hip replacement.

Early plain x-ray changes in Perthes


Disease:
Widening of the joint space.
Sub chondral linear lucency.


 Paediatric orthopaedics

Diagnosis Mode of presentation Treatment Radiology


Developmental Usually diagnosed in Splints and harnesses or Initially no obvious
dysplasia of theinfancy by screening traction. In later years change on plain
hip tests. May be bilateral, osteotomy and hip films and USS gives
when disease is realignment procedures best resolution until
unilateral there may be may be needed. In 3 months of age. On
leg length inequality. arthritis a joint plain films Shentons
As disease progresses replacement may be line should form a
child may limp and needed. However, this is smooth arc
then early onset best deferred if possible
arthritis. More as it will almost
common in extended certainly require
breech babies. revision
Perthes Disease Hip pain (may be Remove pressure from X-rays will show
referred to the knee) joint to allow normal flattened femoral
usually occurring development. head. Eventually in
between 5 and 12 years Physiotherapy. Usually untreated cases the
of age. Bilateral self-limiting if femoral head will
disease in 20%. diagnosed and treated fragment.
promptly.
Slipped upper Typically seen in Bed rest and non-weight X-rays will show the
femoral obese male bearing. Aim to avoid femoral head
epiphysis adolescents. Pain is avascular necrosis. If displaced and falling
often referred to the severe slippage or risk inferolaterally (like
knee. Limitation to of it occurring then a melting ice cream
internal rotation is percutaneous pinning of cone) The
usually seen. Knee the hip may be required. Southwick angle
pain is usually present gives indication of
2 months prior to hip disease severity
slipping. Bilateral in
20%.

Theme: Paediatric fractures

A. Non accidental injury


B. Accidental fracture
C. Rickets
D. Metabolic bone disease of prematurity
E. Hypophosphataemic rickets
F. Osteopetrosis
G. Osteogenesis imperfecta
H. Hypoparathyroidism
I. Osteoporosis

Please select the most likely explanation for each of the following injury scenarios.
Each option may be used once, more than once or not at all.

55. A toddler aged 3 years presents to the Emergency Department with swelling of
his leg and is found to have a spiral fracture of the tibia. His mother reports
that he had tripped and fallen the previous day but she had not noticed any sign
of injury at the time. She is a single parent with little family support. The child
is not on the child protection register.

You answered Rickets

The correct answer is Non accidental injury

Delayed presentation is unusual and should raise concern. In addition spiral


fractures are usually the result of rotational injury which is not compatible with
the mechanism proposed by the parent.

56. A 5 month baby boy presents with swelling of his right arm and is found to
have a spiral fracture of the humerus. He had been in the care of her mother's
boyfriend who reported that he had nearly dropped her that day when reaching
for his bottle and had inadvertently pulled on his arm to save him. He was
immediately taken to the Emergency Department.

Accidental fracture

The mechanism fits with the fracture pattern and the presentation is not
delayed.

57. An infant is admitted with symptoms and signs of respiratory infection and is
found to have several posterior rib fractures on chest radiograph. He was born
prematurely at 37 weeks' gestation and was observed overnight on the special
care baby unit for tachypnoea which settled by the following day. On
assessment it is also apparent that his head circumference has increased at an
excessive rate and has crossed 3 centiles since birth.

You answered Metabolic bone disease of prematurity

The correct answer is Non accidental injury

Posterior rib fractures are extremely unusual in neonates. The change in head
size may be accounted for by hydrocephalus which may occur as a sequelae
from head injury.

Paediatric fractures

Paediatric fracture types


Type Injury pattern
Complete fracture Both sides of cortex are breached
Toddlers fracture Oblique tibial fracture in infants
Plastic deformity Stress on bone resulting in deformity without cortical disruption
Greenstick fracture Unilateral cortical breach only
Buckle fracture Incomplete cortical disruption resulting in periosteal haematoma
only

Growth plate fractures


In paediatric practice fractures may also involve the growth plate and these injuries
are classified according to the Salter- Harris system (given below):

Type Injury pattern


I Fracture through the physis only (x-ray often normal)
II Fracture through the physis and metaphysis
III Fracture through the physis and epiphyisis to include the joint
IV Fracture involving the physis, metaphysis and epiphysis
V Crush injury involving the physis (x-ray may resemble type I, and appear
normal)

As a general rule it is safer to assume that growth plate tenderness is indicative of an


underlying fracture even if the x-ray appears normal. Injuries of Types III, IV and V
will usually require surgery. Type V injuries are often associated with disruption to
growth.

Non accidental injury

 Delayed presentation
 Delay in attaining milestones
 Lack of concordance between proposed and actual mechanism of injury
 Multiple injuries
 Injuries at sites not commonly exposed to trauma
 Children on the at risk register

Pathological fractures
Genetic conditions, such as osteogenesis imperfecta, may cause pathological
fractures.

Osteogenesis imperfecta

 Defective osteoid formation due to congenital inability to produce adequate


intercellular substances like osteoid, collagen and dentine.
 Failure of maturation of collagen in all the connective tissues.
 Radiology may show translucent bones, multiple fractures, particularly of the
long bones, wormian bones (irregular patches of ossification) and a trefoil
pelvis.

Subtypes

 Type I The collagen is normal quality but insufficient quantity.


 Type II- Poor collagen quantity and quality.
 Type III- Collagen poorly formed. Normal quantity.
 Type IV- Sufficient collagen quantity but poor quality.

Osteopetrosis

 Bones become harder and more dense.


 Autosomal recessive condition.
 It is commonest in young adults.
 Radiology reveals a lack of differentiation between the cortex and the medulla
described as marble bone.

In paediatric orthopaedic surgery, which of the following does not fulfill the Kocher
criteria for septic arthritis?

A. ESR > 40mm/h

B. Positive blood culture

C. Fever

D. White cell count > 12, 000


E. Non weight bearing on the affected side

Kocher criteria
1. Non weight bearing on affected side
2. ESR > 40 mm/hr
3. Fever
4. WBC count of >12,000 mm3
- When 4/4 criteria are met, there is a 99% chance that the child has septic arthritis

The Kocher criteria do not consider blood culture results.

Septic arthritis- Paediatric

Septic arthritis

 Staph aureus commonest organism


 Urgent washout and antibiotics otherwise high risk of joint destruction

Diagnosis

 Plain x-rays
 Consider aspiration

Kocher criteria:
1. Non weight bearing on affected side
2. ESR > 40 mm/hr
3. Fever
4. WBC count of >12,000 mm3
- when 4/4 criteria are met, there is a 99% chance that the child has septic arthritis
Theme: Ankle fractures

A. Surgical fixation
B. Below knee amputation
C. Aircast boot
D. Application of full leg plaster cast to include midfoot
E. Application of below knee plaster cast to include the midfoot
F. Application of external fixation device
G. Application of compression bandage and physiotherapy.

Please select the most appropriate management for the injury type described. Each
option may be used once, more than once or not at all.

59. A 24 year old man falls sustaining an inversion injury to his ankle. On
examination he is tender over the lateral malleolus only. On x-ray there is a
fibular fracture that is distal to the syndesmosis.

Application of below knee plaster cast to include the midfoot

Theme from 2008 Exam


These distal injuries are generally managed conservatively. Conservative
management will involve a below knee cast, this will need to extend to the
midfoot. It can be substituted for an aircast boot once radiological union is
achieved.

60. An 86 year old lady stumbles and falls whilst opening her front door. On
examination her ankle is swollen with both medial and lateral tenderness. X
rays demonstrate a fibular fracture at the level of the syndesmosis.

You answered Application of external fixation device

The correct answer is Application of below knee plaster cast to include the
midfoot

Although, this is a potentially unstable injury operative fixation in this age


group generally gives poor results owing to poor quality bone. A below knee
cast should be applied in the first instance. If this fails to provide adequate
control it can be extended above the knee.

61. A 25 year old man suffers an injury whilst playing rugby involving a violent
twist to his left lower leg. On examination both malleoli are tender and the
ankle joint is very swollen. On x-ray there is a spiral fracture of the fibula and
widening of the ankle mortise.

Surgical fixation

This is a variant of the Weber C fracture in which disruption of the tibio-


fibular syndesmosis occurs leading to joint disruption. Surgical repair is
warranted.

Ankle injuries

Ankle fractures are a common cause of admission to casualty. Clinical examination is


facilitated by the Ottawa ankle rules to try and minimise the unnecessary use of x-
rays.

 These state that x-rays are only necessary if there is pain in the malleolar zone
and:
1. Inability to weight bear for 4 steps
2. Tenderness over the distal tibia
3. Bone tenderness over the distal fibula

A number of classification systems exist for describing ankle fractures, these include
the Potts, Weber and AO systems. For simplicity the Weber system is outlined here.

Weber classification
Related to the level of the fibular fracture.

 Type A is below the syndesmosis


 Type B fractures start at the level of the tibial plafond and may extend
proximally to involve the syndesmosis
 Type C is above the syndesmosis which may itself be damaged

A subtype known as a Maisonneuve fracture may occur with spiral tibial fracture
that leads to disruption of the syndesmosis with widening of the ankle joint, surgery is
required.

Management
Depends upon stability of ankle joint and patient co-morbidites.
All ankle fractures should be promptly reduced to remove pressure on the overlying
skin and subsequent necrosis.
Young patients, with unstable, high velocity or proximal injuries will usually require
surgical repair. Often using a compression plate.
Elderly patients, even with potentially unstable injuries usually fare better with
attempts at conservative management as their thin bone does not hold metalwork well.
Theme: Management of hip fractures

A. Hemiarthroplasty cemented prosthesis


B. Hemiarthroplasty non cemented prosthesis
C. Percutaneous pinning
D. Conservative management
E. Dynamic hip screw
F. Intramedullary device
G. Hip arthrodesis
H. Total hip replacement

For each fracture scenario please select the most appropriate management option from
the list. Each option may be used once, more than once or not at all.

62. A 72 year old retired teacher is admitted to A&E with a fall and hip pain. He is
normally fit and well. He lives with his son in a detached, 2 storey house. A
hip x-ray confirms an extracapsular fracture.
You answered Conservative management

The correct answer is Dynamic hip screw

Extracapsular fractures should be treated surgically. Since the blood supply to


the femoral head is not compromised joint replacement is not usually
warranted.

63. A 72 year old retired teacher is admitted to A&E with a fall and hip pain. He is
normally fit and well. He lives with his son in a detached, 2 storey house. A
hip x-ray confirms an subtrochanteric fracture.

You answered Dynamic hip screw

The correct answer is Intramedullary device

Intramedullary device is normally recommended for reverse oblique,


transverse or subtrochanteric fractures.

64. An 86 year old retired pharmacist is admitted to A&E following a fall. She
complains of right hip pain. She is known to have hypertension and is
currently on bendrofluazide. She lives alone and does not mobilise. Her right
leg is shortened and externally rotated. A hip x-ray confirms a displaced
intracapsular fracture.

You answered Total hip replacement

The correct answer is Hemiarthroplasty non cemented prosthesis

This patient warrants a hemiarthroplasty due to reduced mobility and older


age. The anterolateral approach is recommended in the SIGN guidelines. In
this case most surgeons would not use a cemented prosthesis.

Hip fractures

The hip is a common site of fracture especially in osteoporotic, elderly females. The
blood supply to the femoral head runs up the neck and thus avascular necrosis is a risk
in displaced fractures.

Classification
The Garden system is one classification system in common use.
 Type I: Stable fracture with impaction in valgus.
 Type II: Complete fracture but undisplaced.
 Type III: Displaced fracture, usually rotated and angulated, but still has bony
contact.
 Type IV: Complete bony disruption.

Blood supply disruption is most common following Types III and IV.

Management of hip fractures in older adults


SIGN Guidelines
Fracture type Patient co-morbidities Management
Undisplaced Nil Internal fixation (especially if
intracapsular fracture young)
Undisplaced Major illness or Hemiarthroplasty
intracapsular fracture advanced organ specific
disease
Displaced intracapsular Nil If age <70 then internal fixation (if
fracture possible), hip arthroplasty if not
Displaced intracapsular Nil Age >70 total hip arthroplasty
fracture
Displaced intracapsular Major/ immobile Hemiarthroplasty
fracture
Extracapsular fracture Only major co- Dynamic hip screw
(non special type) morbidities affect
management
Extracapsular fracture Only major co- Usually intramedullary device
(reverse oblique, morbidities affect
transverse or sub management
trochanteric)

A typical image of an intracapsular fracture occurring in an elderly osteoporotic lady


Image sourced from Wikipedia

References
www.sign.ac.uk/guidelines/fulltext/111/index.html
Theme: Paediatric orthopaedics

A. USS hip
B. Hip x-ray
C. Anteroposterior pelvic x-ray
D. CT scan
E. MRI scan
F. Technetium bone scan
G. USS knee
H. X-ray knee
I. Discharge and reassure

For each of the following scenarios which is the most appropriate investigation? Each
option may be used once, more than once or not at all.

65. An obese 12 year old boy presents with knee pain. On examination he has pain
on internal rotation of the hip. His knee is clinically normal.

Hip x-ray

The main differential diagnosis in a boy over 10 years old is of slipped upper
femoral epiphysis. Knee pain is a common presenting feature. An
anteroposterior pelvic x-ray may miss a minor slip, therefore request a hip
film.
66. A baby is delivered in the breech position. Barlows and Ortolani tests are
normal

You answered Discharge and reassure

The correct answer is USS hip

This child is at risk of developmental dysplasia of the hip (up to 20% will have
DDH), so should have the hip joints scanned to exclude this.

67. A 5 year old boy presents with a painful limp. The symptoms have been
present for 8 weeks. Two hip x-rays have been performed and appear normal.

You answered USS hip

The correct answer is Technetium bone scan

Perthes disease should be suspected in boys over 4 years old presenting with a
limp. Early disease can be missed on x-ray, therefore a bone scan should be
performed. MRI is less sensitive than the bone scan.

Paediatric orthopaedics

Diagnosis Mode of presentation Treatment Radiology


Developmental Usually diagnosed in Splints and harnesses or Initially no obvious
dysplasia of theinfancy by screening traction. In later years change on plain
hip tests. May be bilateral, osteotomy and hip films and USS gives
when disease is realignment procedures best resolution until
unilateral there may be may be needed. In 3 months of age. On
leg length inequality. arthritis a joint plain films Shentons
As disease progresses replacement may be line should form a
child may limp and needed. However, this is smooth arc
then early onset best deferred if possible
arthritis. More as it will almost
common in extended certainly require
breech babies. revision
Perthes Disease Hip pain (may be Remove pressure from X-rays will show
referred to the knee) joint to allow normal flattened femoral
usually occurring development. head. Eventually in
between 5 and 12 years Physiotherapy. Usually untreated cases the
of age. Bilateral self-limiting if femoral head will
disease in 20%. diagnosed and treated fragment.
promptly.
Slipped upper Typically seen in Bed rest and non-weight X-rays will show the
femoral obese male bearing. Aim to avoid femoral head
epiphysis adolescents. Pain is avascular necrosis. If displaced and falling
often referred to the severe slippage or risk inferolaterally (like
knee. Limitation to of it occurring then a melting ice cream
internal rotation is percutaneous pinning of cone) The
usually seen. Knee the hip may be required. Southwick angle
pain is usually present gives indication of
2 months prior to hip disease severity
slipping. Bilateral in
20%.

A 5 year old boy is playing in a tree when he falls and lands on his right forearm. He
is brought to the emergency department by his parents. On examination he has bony
tenderness and bruising. An X-ray is taken and shows unilateral cortical disruption is
development of periosteal haematoma. Which of the following is the most likely
diagnosis?

A. Buckle fracture

B. Greenstick fracture

C. Toddlers fracture

D. Complete fracture

E. None of the above

Greenstick fractures are common childhood injuries. Unilateral cortical disruption is


the main radiological feature, since involvement of both cortices makes the injury a
complete fracture. Buckle fractures will show periosteal haematoma formation only.

Paediatric fractures

Paediatric fracture types


Type Injury pattern
Complete fracture Both sides of cortex are breached
Toddlers fracture Oblique tibial fracture in infants
Plastic deformity Stress on bone resulting in deformity without cortical disruption
Greenstick fracture Unilateral cortical breach only
Buckle fracture Incomplete cortical disruption resulting in periosteal haematoma
only

Growth plate fractures


In paediatric practice fractures may also involve the growth plate and these injuries
are classified according to the Salter- Harris system (given below):

Type Injury pattern


I Fracture through the physis only (x-ray often normal)
II Fracture through the physis and metaphysis
III Fracture through the physis and epiphyisis to include the joint
IV Fracture involving the physis, metaphysis and epiphysis
V Crush injury involving the physis (x-ray may resemble type I, and appear
normal)

As a general rule it is safer to assume that growth plate tenderness is indicative of an


underlying fracture even if the x-ray appears normal. Injuries of Types III, IV and V
will usually require surgery. Type V injuries are often associated with disruption to
growth.

Non accidental injury

 Delayed presentation
 Delay in attaining milestones
 Lack of concordance between proposed and actual mechanism of injury
 Multiple injuries
 Injuries at sites not commonly exposed to trauma
 Children on the at risk register

Pathological fractures
Genetic conditions, such as osteogenesis imperfecta, may cause pathological
fractures.

Osteogenesis imperfecta

 Defective osteoid formation due to congenital inability to produce adequate


intercellular substances like osteoid, collagen and dentine.
 Failure of maturation of collagen in all the connective tissues.
 Radiology may show translucent bones, multiple fractures, particularly of the
long bones, wormian bones (irregular patches of ossification) and a trefoil
pelvis.

Subtypes

 Type I The collagen is normal quality but insufficient quantity.


 Type II- Poor collagen quantity and quality.
 Type III- Collagen poorly formed. Normal quantity.
 Type IV- Sufficient collagen quantity but poor quality.

Osteopetrosis

 Bones become harder and more dense.


 Autosomal recessive condition.
 It is commonest in young adults.
 Radiology reveals a lack of differentiation between the cortex and the medulla
described as marble bone.

Theme: Eponymous fractures

A. Smith's
B. Bennett's
C. Monteggia's
D. Colle's
E. Galeazzi
F. Pott's
G. Barton's

Which is the most likely eponymous fracture for the scenario given. Each option may
be used once, more than once or not at all.

69. A 14 year old boy jumps off a 10 foot wall and lands on both feet. An x-ray
shows a bimalleolar fracture of the right ankle.

Pott's

70. A 22 year old drunk man is involved in a fight. He hurts his thumb when he
punches his opponent.

Bennett's

71. A 63 year nurse falls on an extended and pronated wrist. An x-ray shows a
distal radial fracture with radiocarpal dislocation.

Barton's

Eponymous fractures

Colles' fracture (dinner fork deformity)

 Fall onto extended outstretched hand


 Classical Colles' fractures have the following 3 features:
1. Transverse fracture of the radius
2. 1 inch proximal to the radio-carpal joint
3. Dorsal displacement and angulation

Smith's fracture (reverse Colles' fracture)

 Volar angulation of distal radius fragment (Garden spade deformity)


 Caused by falling backwards onto the palm of an outstretched hand or falling
with wrists flexed

Bennett's fracture

 Intra-articular fracture of the first carpometacarpal joint


 Impact on flexed metacarpal, caused by fist fights
 X-ray: triangular fragment at ulnar base of metacarpal

Image sourced from Wikipedia


Monteggia's fracture

 Dislocation of the proximal radioulnar joint in association with an ulna


fracture
 Fall on outstretched hand with forced pronation
 Needs prompt diagnosis to avoid disability
Image sourced from Wikipedia
Galeazzi fracture

 Radial shaft fracture with associated dislocation of the distal radioulnar joint
 Direct blow

Pott's fracture

 Bimalleolar ankle fracture


 Forced foot eversion

Barton's fracture

 Distal radius fracture (Colles'/Smith's) with associated radiocarpal dislocation


 Fall onto extended and pronated wrist

Theme: Fracture management

A. Application of external fixator


B. Open reduction and internal fixation
C. Fasciotomy
D. Skeletal traction
For the following upper limb injuries please select the most appropriate initial
management. Each option may be used once, more than once or not at all.

72. A 32 year old man falls from a ladder and sustains a fracture of his proximal
radius. On examination he has severe pain in his forearm and diminished distal
sensation. There is a single puncture wound present at the fracture site.

Fasciotomy

Theme from April 2012 Exam


Pain and neurological symptoms in a tight fascial compartment coupled with a
high velocity injury carry a high risk of compartment syndrome and prompt
fasciotomy should be performed.

73. A 32 year old man falls a sustains a fracture of his distal humerus. The fracture
segment is markedly angulated and unstable. There is a puncture site overlying
the fracture site.

You answered Open reduction and internal fixation

The correct answer is Application of external fixator

Wide exposure to plate the humerus is generally inadvisable owing to its many
important anatomical relations. Both intramedullary nailing and external
fixation are reasonable treatments. However, in the presence of an open
fracture application of an external fixator and appropriate tissue debridement
would be most appropriate.

74. A 24 year old man sustains a distal radius fracture during a game of rugby.
Imaging shows a comminuted fracture with involvement of the articular
surface.

You answered Application of external fixator

The correct answer is Open reduction and internal fixation

Meticulous anatomical alignment of the fracture segments is crucial to avoid


the development of osteoarthritis and risk of malunion.

Fracture management
 Bony injury resulting in a fracture may arise from trauma (excessive forces
applied to bone), stress related (repetitive low velocity injury) or pathological
(abnormal bone which fractures during normal use of following minimal
trauma)
 Diagnosis involves not just evaluating the fracture ; such as site and type of
injury but also other associated injuries and distal neurovascular deficits. This
may entail not just clinical examination but radiographs of proximal and distal
joints.
 When assessing x-rays it is important to assess for changes in length of the
bone, the angulation of the distal bone, rotational effects, presence of material
such as glass.

Fracture types
Fracture type Description
Oblique fracture Fracture lies obliquely to long axis of bone
Comminuted fracture >2 fragments
Segmental fracture More than one fracture along a bone
Transverse fracture Perpendicular to long axis of bone
Spiral fracture Severe oblique fracture with rotation along long axis of bone

Open Vs Closed
It is also important to distinguish open from closed injuries. The most common
classification system for open fractures is the Gustilo and Anderson classification
system (given below):

Grade Injury
1 Low energy wound <1cm
2 Greater than 1cm wound with moderate soft tissue damage
3 High energy wound > 1cm with extensive soft tissue damage
3 A (sub group of 3) Adequate soft tissue coverage
3 B (sub group of 3) Inadequate soft tissue coverage
3 C (sub group of 3) Associated arterial injury

Key points in management of fractures

 Immobilise the fracture including the proximal and distal joints


 Carefully monitor and document neurovascular status, particularly following
reduction and immobilisation
 Manage infection including tetanus prophylaxis
 IV broad spectrum antibiotics for open injuries
 As a general principle all open fractures should be thoroughly debrided ( and
internal fixation devices avoided or used with extreme caution)
 Open fractures constitute an emergency and should be debrided and lavaged
within 6 hours of injury

Theme: Fracture management


A. Copious lavage and generous surgical debridement, followed by
external fixation
B. Intramedullary nail
C. Open reduction and internal fixation
D. Immobilisation in plaster cast
E. External fixation using a frame device
F. Amputation
G. Application of external fixation device
H. Primary closure of wound and application of plaster cast

Please select the most appropriate management for the fractures described. Each
option may be used once, more than once or not at all.

75. A 55 year old motorcyclist is involved in a road traffic accident and sustained
a Gustilo and Anderson IIIc type fracture to the distal tibia. He was trapped in
the wreckage for 7 hours during which time he bled profusely from the fracture
site. He has an established distal neurovascular deficit.

You answered Copious lavage and generous surgical debridement, followed


by external fixation

The correct answer is Amputation

This man is unstable, and at 7 hours after extraction, the limb is not viable. The
safest option is primary amputation.

76. A 25 year old ski instructor who falls off a ski lift and sustains a spiral fracture
of the mid shaft of the tibia. Attempts to achieve satisfactory position in plaster
have failed. Overlying tissues are healthy.

Intramedullary nail

This would be a good case for intramedullary nailing. Open reduction and
external fixation would strip off otherwise healthy tissues and hence is
unsuitable. In some units the injury may be managed with an Ilizarov frame
device but the majority would treat with IM nailing.

77. A 35 year old mechanic is hit by a fork lift truck. He sustains a Gustilo and
Anderson type IIIA fracture of the shaft of the left femur.

You answered External fixation using a frame device

The correct answer is Copious lavage and generous surgical debridement,


followed by external fixation
At the tissues are in better shape than in the first case and as there is no
associated vascular injury the patient may be suitable for debridement of the
area and external fixation. If debridement leaves a tissue defect then plastic
surgical repair will be needed at a later stage.

Delayed treatment of open fractures with


significant vascular injury may be best
treated by primary amputation.

Fracture management

 Bony injury resulting in a fracture may arise from trauma (excessive forces
applied to bone), stress related (repetitive low velocity injury) or pathological
(abnormal bone which fractures during normal use of following minimal
trauma)
 Diagnosis involves not just evaluating the fracture ; such as site and type of
injury but also other associated injuries and distal neurovascular deficits. This
may entail not just clinical examination but radiographs of proximal and distal
joints.
 When assessing x-rays it is important to assess for changes in length of the
bone, the angulation of the distal bone, rotational effects, presence of material
such as glass.

Fracture types
Fracture type Description
Oblique fracture Fracture lies obliquely to long axis of bone
Comminuted fracture >2 fragments
Segmental fracture More than one fracture along a bone
Transverse fracture Perpendicular to long axis of bone
Spiral fracture Severe oblique fracture with rotation along long axis of bone

Open Vs Closed
It is also important to distinguish open from closed injuries. The most common
classification system for open fractures is the Gustilo and Anderson classification
system (given below):

Grade Injury
1 Low energy wound <1cm
2 Greater than 1cm wound with moderate soft tissue damage
3 High energy wound > 1cm with extensive soft tissue damage
3 A (sub group of 3) Adequate soft tissue coverage
3 B (sub group of 3) Inadequate soft tissue coverage
3 C (sub group of 3) Associated arterial injury

Key points in management of fractures

 Immobilise the fracture including the proximal and distal joints


 Carefully monitor and document neurovascular status, particularly following
reduction and immobilisation
 Manage infection including tetanus prophylaxis
 IV broad spectrum antibiotics for open injuries
 As a general principle all open fractures should be thoroughly debrided ( and
internal fixation devices avoided or used with extreme caution)
 Open fractures constitute an emergency and should be debrided and lavaged
within 6 hours of injury

You might also like